You are on page 1of 102

MATH BOOK

FUNCTION
Textbook booklet with Theories &
Exercises

Short Book
JEE MAIN | CBSE

WITH SOLUTION BOOK

BY APARNA DWIVEDI
MATH BOOK

FUNCTION

Short Book
JEE Main | CBSE

BY APARNA DWIVEDI
Page # 2 FUNCTION

Content

S.NO. TOPIC PAGE NO.

♦ THEORY WITH SOLVED EXAMPLES .............................................................. 3 – 27

♦ EXERCISE - I .................................................................................................. 28 – 39

♦ EXERCISE - II .................................................................................................. 40 – 42

♦ EXERCISE - III ................................................................................................. 43 – 54

♦ EXERCISE - IV ................................................................................................ 55 – 65

♦ EXERCISE - V ................................................................................................. 66 – 68

♦ ANSWER KEY ................................................................................................ 69 – 72

JEE Syllabus :

Real valued functions of a real variable, into, onto and one-to-one functions, sum, difference, product
and quotient of two functions, composite functions, absolute value, polynomial, rational, trigonometric,
exponential and logarithmic functions,
FUNCTION Page # 3

A. DEFINITION

Function is defined as a rule or a manner or a mapping or a correspondence f which maps each & every
element of set A with a unique element of set B. It is denoted by :
f
f:A→B or A → B we read it as “ f is a function from A to B”

f f
A B A B
1 w w
x 1 x
Ex. f maps A to B 2
3 y Yes y No
2
4 z z

Figure (1) Figure (2)


f
A B
1 w
Figure –2 does not represent a function because conversion 2
3 Yes
is allowed (figure–3) But diversion is not allowed. x
4

Figure (3)
Ex.1 Which of the following correspondences can be called a function ?

(A) f(x) = x3 ; {–1, 0, 1} → {0, 1, 2, 3} (B) f(x) =± x ; {0, 1, 2} → {–2, –1, 0, 1, 2}

(C) f(x) = x ; {0, 1, 4} → {–2, –1, 0, 1, 2} (D) f(x) = – x ; {0, 1, 4} → {–2, –1, 0, 1, 2}
Sol. f(x) in (C) & (D) are functions as definition of function is satisfied. while in case of (A) the given
relation is not a function, as f(–1) ∉ codomain. Hence definition of function is not satisfied. While in
case of (B), the given relation is not a function, as f(1) = ± 1 and f(4) = ± 2 i.e. element 1 as well as
4 in domain are related with two elements of codomain. Hence definition of function is not satisfied.

Ex.2 If X = {a, b, c, d, e} & Y = {p, q, r, s, t} then which of the following subset(s) of X × Y is/are a
function from X to Y.
(A) {(a, r) (b, r) (b, s) (d, t) (e, q) (c, q)} (B) {(a, r) (b, p) (c, t) (d, q)}
(C) {(a, p) (b, t) (c, r) (d, s) (e, q)} (D) {(a, r) (b, r) (c, r) (d, r) (e, r)}
Sol. Let us check every option for the two conditions of the function
(A) ∵ b has two output (images) namely r & s ∴ Not a function
(B) ∵ e ∈ X does not have any image ∴ Not a function
(C) ∵ every element of X has one and only one output ∴ it is a function
(D) ∵ every element’s output is r ∴ it is a function
Hence correct options are (C) & (D).

“Function” as an ordered pair : f


f:A→B A B
1
f : {(1, x), (2, x), (3, x)} x
2
⇒ f⊂A×B
3 y
where A × B is the cartesian product of two set A & B
= {(1, x), (2, x), (3, x), (1, y), (2, y), (3, y)}
Page # 4 FUNCTION

Remark : Every function from A to B satisfied the following Relation :


(1) f⊂A×B
(2) Notation : ∀ a ∈ A ∃ b ∈ B | b = f(a)
↓ ↓ ↓
(For all) (there exist) (such that)
(3) (a, b) ∈ f & (a, c) ∈ f ⇒ b = c
(4) In a graphical representation of a function y = f(x).
If vertical line cuts the curve more than once then it is not a function. It is called as vertical line test

y2 = 4ax y = f(x)

Ex : It is not a function as vertical line touches

curve at more than once point.

B. DOMAIN, CO-DOMAIN & RANGE OF A FUNCTION

Let f : A → B, then the set A is known as the domain of f& the set B is known as co-domain of f. If a
member ‘a’ of A is associated to the member ‘b’ of B, then’b’ is called the f-image of ‘a’ and we write
b = f(a). Further ‘a’ is called a pre-image of ‘b’. The set {f(a): ∀ a ∈ A} is called the range of f and
is denoted by f(A). Clearly f(A) ⊆ B.
If only expression of f(x) is given (domain and codomain are not mentioned), then domain is set of those
values of ‘x’ for which f (x) is real, while codomain is considered to be (–∞, ∞) (except in ITFs)
A function whose domain and range are both subsets of real numbers is called a real function.

(Algebraic Operations on Functions) : If f & g are real valued functions of x with domains A and B
respectively, then both f & g are defined in A ∩ B . Now we define f + g , f − g , (f . g) & (f/g) as follows:
(i) (f ± g) (x) = f(x) ± g(x)

(ii) (f . g) (x) = f(x) . g(x)  domain in each case is A ∩ B

f
(iii)   (x) = f( x ) domain is {x  x ∈ A ∩ B and g(x) ≠ 0} .
 g g( x )

x−5
Ex.3 Find the domain of definition of the function y = log10 −3 x+5
x − 10 x + 24
2

Sol. For y to be defined

x−5
(i) 2
>0
x − 10x + 24
When x – 5, x = 5 and when x2 – 10 x + 24, x = 4, 6
x−5
sign scheme for is as follows. ∞
x 2 − 10 x + 24 –∞ –ve 4 +ve 5 –ve 6 +ve

x−5
Put x = 0 ∴ > 0 ⇒ 4 < x < 5 or x > 6 ...(A)
2
x − 10x + 24
1
(ii) ( x + 5) is defined for all x
3 ...(B)

Combining (A) and (B), we get 6 < x < ∞ or 4 < x < 5 ∴ Domain = ] 4, 5 [ U ] 6, ∞ [
FUNCTION Page # 5

Ex.4 Find the domain of the function f(x) = x2 − x − 6 + 6−x

Sol. x2 – x – 6 ≥ 0 and 6–x≥0


(x – 3) (x + 2) ≥ 0 x≤6
x ≥ 3 or x ≤ –2 x ∈ (–∞, 6]
x ∈ (–∞, –2] ∪ [3, ∞)
–2 –3 6

∴ x ∈ (–∞, –2] ∪ [3, 6]

Ex.5 Find the domain of definition of the function,

1 − 5x
(a) f (x) = (x 2
− 3 x − 10) .  n (x − 3)
2
(b)
7− x − 7
Sol. (a) (x − 5) (x + 2) n2 (x − 3) ≥ 0

Equality gives x = 5 ; x = 4 (Note for x = − 2 n (x − 3) is meaningless)


Now (x − 5) (x + 2) > 0 and x − 3 > 0 ⇒ x > 5 hence the result
Ans. : {4} ∪ [5, ∞)
(b) 7−x ≠ 7 ⇒ x ≠ −1

1 − 5x 1 − 5x
Now =0 ⇒ x=0 ; Now solve >0
7 −x
−7 7− x − 7

1 − 5x > 0 and 7–x − 7 > 0 or 1 − 5x < 0 and 7–x − 7 < 0


Ans. : (− ∞, − 1) ∪ [0, ∞)

x−7 1
Ex.6 Find the Domain of the function f(x) = –
3−x 3 2
x − 6x + 8

x−7 x−7
Sol. ≥0 ⇒ ≤0 ⇒ x ∈ (3, 7]
3−x x−3

& x2 – 6x + 8 ≠ 0 ⇒ (x – 4) (x – 2) ≠ 0 ⇒ x ≠ 2, 4
Domain x ∈ (3, 7] – {4}

Ex.7 Find the domain of given function f(x) = 3x − x3

Sol. 3x – x3 ≥ 0 ⇒ x3 – 3x ≤ 0 ⇒ x(x2 – 3) ≤ 0 ⇒ x(x – 3 ) (x + 3)≤0

– + – +
∴ x ∈ (− ∞, − 3 ] ∪ [0, 3]
− 3 3
Page # 6 FUNCTION

C. IMPORTANT TYPE OF FUNCTIONS


(1) Trigonometric function :
Function Domain Range Curve
y = sin x
1
−π / 2 3π / 2
(i) f(x) = sin x x∈R y ∈ [–1, 1] −2 π −3π / 2 −π π/2 π 2π
x

–1

y = cos x
1
−π / 2 3π / 2
−2π −3π / 2 −π π/ 2 π 2π 5π / 2
x
(ii) f(x) = cos x x∈R y ∈ [–1, 1]
–1

y = tan x

π π π x

3π −π−2 O 3π
π 2 2 2
(iii) f(x) = tan x x ∈ R – (2n + 1) , n ∈ Ι y∈R
2

y = cot x


2
(iv) f(x) = cot x x ∈ R – nπ, n ∈ Ι y∈R −π

π O π π 2π
x
2 2

y = cosec x

−π
−π / 2 0 π/2
π 2π
x
(v) f(x) = cosec x x ∈ R –nπ, n∈Ι y ∈ (−∞, –1] ∪ [1, ∞) 3π / 2
–1

y = sec x

1
−π π
x
π −3π / 2 −π / 2 0 π/2 3π / 2
(vi) f(x) = sec x x∈R – (2n+1) , n∈Ι y ∈ (−∞, –1] ∪ [1, ∞)
2 –1
FUNCTION Page # 7

(2) Polynomial Function :


f(x) = a0xn + a1xn – 1 + a2 xn–2 +.......+ an
where a0, a1, a2 ........ an ∈ R n∈W
If a0 ≠ 0, then f(x) is called n degree polynomial and Domain x ∈ R
th

(3) Algebraic Function : A function is called an algebraic function. If it can be constructed using
algebraic operations such as additions, subtractions, multiplication, division taking roots etc.
All polynomial functions are algebraic but converse is not true.

Ex : f(x) = x 4 + 5x 2 + x + (x + 5) , f(x) = x + 3x + x + 5
3 3/5 3 2

Remark : Function which are not algebraic are called as TRANSCENDENTAL FUNCTION.

( x 5 + 5 x 2 )3 / 5
Ex : f(x) = + 3 x 2 + 5 x + 6 + n x → transcidental function
x3

x+7
Ex : f(x) = x 2 + 7 + en x + → algebraic function.
x2 + 7

g( x )
(4) Rational Function : It is a function of form f(x) = , where g(x) & h(x) are poly. function
h( x )

x 4 − 3x 2 + 2
and h(x) ≠ 0 Ex. f(x) =
x2 − 4

(5) Logarithmic function : f(x) = logax, where x > 0, a > 0, a ≠ 1


a → base, x → number or argument of log.
Case–I : 0 < a < 1 Case–II : a > 1
f(x)
f(x)
f(x) = loga x f(x) = n x
Domain : x ∈ (0, ∞) (1, 0)
O (1, 0)
O x
x
Range : y ∈ R

(6) Exponential function : f(x) = ax, where a > 0, a ≠ 1


a → Base x → Exponent
Case–I : 0 < a < 1 ; a = 1/2 Case–II : a > 1

x f(x) f(x)
 1
f(x) =  
2 (1, 0)
(0,1)
Domain : x ∈ R
O x O x
Range : y ∈ (0, ∞)
Page # 8 FUNCTION

(7) Absolute value function (Modulus function) : y

 x ; x≥0 y=x
y = |x| =  − x ; x < 0 y=–x

x
Domain : x ∈ R; Range : y ∈ R+ ∪ {0}

(8) Signum function : y

1
 1 ; x>0

y = sgn (x) =  0 ; x = 0 0 x
 − 1 ; x < 0 –1

Domain : x ∈ R; Range : y ∈ {–1, 0, 1}

(9) Greatest integer function (step-up function) :


y

 x ; x ∈I 2

y = f(x) = [x]  Greatest Integer ; otherwise
1
 less than x

0
Domain : x ∈ R; Range : y ∈ I 1 2 3 x

Ex : [2 . 3] = 2, [5] = 5, [–2 . 3] = –3 –1

Properties :

 0 ; x∈I
(i) [x] ≤ x < [x] + 1 (ii) [x + m] = [x] + m ; m ∈ I (iii) [x] + [–x] = − 1 ; x ∉ I

(10) Fractional part function :


y = f(x) = {x} = x – [x] y

Domain : x ∈ R; Range : [0, 1) 1


Ex : 2.3 = 2 + 0.3 → fractional part
↓ –1 –2 0 1 2 3 x
Integer part
Properties :
(i) Fractional part of any integer is zero. (ii) {x + n} = {x}, n ∈ Ι

(iii) {
0; x∈I
{x} + {–x} = 1 ; otherwise

1  x2 + 1 
 2 
Ex.8 Find the range of the following functions : (a) y =
2 + sin 3 x + cos 3 x
(b) y = sin–1  x + 2
 

1 1
Sol. (a) We have y= i.e. sin 3x + cos 3x =
2 + sin 3 x + cos 3 x y –2

 π 1  π 1  1 − 2 
i.e. 2 sin  3x + 4  = y – 2 i.e. sin  3x +  = 
2 y


 4
FUNCTION Page # 9

 π 1
since, sin 3 x +  ≤ 1, therefore we have − 2 ≤ 2
 4 y

1
i.e. – 2≤1 –2≤ 2 i.e. 2– 2≤ ≤2 2
y y

1 1  1  1
i.e. ≤y≤ Hence, the range is y ∈  ,
.
2+ 2 2− 2 2 + 2 2 − 2 

x2 + 1 1
(b) We have = 1− 2
x +2
2
x +2

1 1 −1 −1
Now, we have 2 ≤ x2 + 2 < ∞ i.e. ≥ 2 >0 i.e. ≤ 2 <0
2 x +2 2 x +2

1 1 1 x2 + 1 1  x2 + 1 
≤ 1− 2 ≤ ≤ sin–1  
i.e. 1–
2 x +2
<1 i.e.
2 x2 + 2
<1 i.e. sin–1  x 2 + 2  < sin–1 1
2  

π π π π 
gives ≤y< Hence, the range is y ∈  , .
6 2 6 2 

Ex.9 Find the range of following functions : (i) y = ln (2x – x2) (ii) y = sec–1 (x2 + 3x + 1)
Sol. (i) using maxima–minima, we have (2x – x ) ∈ (–∞, 1]
2

For log to be defined accepted values are 2x – x2 ∈ (0, 1] {i.e. domain (0, 1]}
ln (2x – x2) ∈ (0, 1] ∴ range is (–∞, 0]
(ii) y = sec–1 (x2 + 3x + 1)
y
 5 
Let t = x2 + 3x + 1 for x ∈ R then t ∈ − , ∞  π
 4 
 5 
t ∈  − ,−1 ∪ [1, ∞)
sec–1 (–5/4)
but y = sec–1 (t) ⇒ π/2
 4 

 π  −1 5 
from graph range is y ∈ 0,  ∪ sec  − , π
t
 2   4  –1
–5/4 0 1

 −1
x 2 + x + 1)
Ex.10 Find the range of y = ln(sin 

2
 1 3
Sol. We have x + x + 1 = x +  +
2
which is a positive quantity whose minimum value is 3/4.
 2  4

 −1
x 2 + x + 1) to be defined, we have x2 + x+ 1 ≤ 1
Also, for the function y = ln(sin 

Page # 10 FUNCTION

3 3 π π
Thus, we have ≤ x2 + x + 1 ≤ 1 i.e. ≤ x2 + x + 1 ≤ i.e. ≤ sin−1( x 2 + x + 1) ≤
4 2 3 2

[∴ sin–1 x is an increasing function, the inequality sign remains same]

 π π
i.e. ln   ≤ ln(sin–1 x 2 + x + 1 ≤ ln  
3 2

i.e. 0.046 ≤ ln(sin–1 x 2 + x + 1 )] = 0. Hence, the range is y ∈ [n π/3, n π/2]

3x 2 + mx + n
Ex.11 f : R → R, f (x) = . If the range of this function is [– 4, 3) then find the value of (m2 + n2).
x2 +1

3( x 2 + 1) + mx + n − 3 mx + n − 3
Sol. f (x) = ; f (x) = 3 +
1+ x 2
1+ x2
mx + n − 3
y=3+ for y to lie in [– 4, 3) mx + n – 3 < 0 ∀ x ∈ R
1+ x2

n −3
this is possible only if m = 0 when, m = 0 then y = 3 +
1+ x2
note that n – 3 < 0 (think !) n<3 i f x → ∞, ymax → 3–
now ymin occurs at x = 0 (as 1 + x2 is minimum)
ymin = 3 + n – 3 = n ⇒ n = – 4 so m2 + n2 = 16

  4 − x 2 
 
Ex.12 Find the domain and range of f(x) = sin n 1 − x 
  

Sol. 4 − x 2 is positive and x < 4 ⇒ –2 < x < 2


2

1 – x should also be positive. ∴ x < 1

 4 − x2 
Thus the domain of n  is –2 < x < 1 sine being defined for all values, the domain of sin
 1− x 
 
  4 − x 2   4 − x2 
   
n  is the same as the domain of  n
 1− x 
  1 − x   
4 − x2
To study the range. Consider the function
1− x
4 − x2 4 − x2
As x varies from –2 to 1, varies in the open interval (0, ∞) and hence n varies from
1− x 1− x
  4 − x2  
  
–∞ to + ∞. Therefore the range of sin  n 1 − x   is (–1, +1)
  
FUNCTION Page # 11

1+ x4
Ex.13 Find the range of the function f(x) = sin–1 .
1 + 5 x10

1+ x4
Sol. Consider g(x) = . Also g(x) is positive ∀ x ∈ R and g(x) is continuous ∀ x ∈ R and
1 + 5 x10

g(0) = 1 and xlim


→∞
g(x) = 0

 π
⇒ g(x) can take all values from (0, 1] ⇒ Range of f(x) = sin–1 (g(x)) is  0,  .
2  

Ex.14 f(x) = cos–1 {log [ [ x 3 + 1] ]}, find the domain and range of f(x) (where [ * ] denotes the greatest

integer function).

Sol. If cos–1 x = θ, then– 1 ≤ x ≤ 1 ∴ –1 ≤ log [ [ x 3 + 1] ] ≤ 1 ⇒ e–1 ≤ [ [ x 3 + 1] ] ≤ e

0.37 ≤ [ [ x 3 + 1] ] ≤ 2.7 ∴ 1≤ [ x 3 + 1] < 3 ⇒ 1 ≤ [x3 + 1] < 9

1 ≤ [x3] + 1 < 9 0 ≤ [x3] < 8 ∴ 0 ≤x < 2


∴ Domain of f(x) = Dr in x ∈ [0, 2) Range of f(x) When 0 ≤ x < 2

Then 1 ≤ x3 + 1 < 9 ∴ 1 ≤ [x3 + 1] ≤ 8 ⇒ 1≤ [ x 3 + 1] ≤ 2 2 ⇒ 1 ≤ [ x 3 + 1] ≤ 2.8

Case I : 1 ≤ [ x 3 + 1] < 2 then [ [ x 3 + 1] ] = 1 Case II : 2 ≤ [ x 3 + 1] ≤ 2.8 then [ [ x 3 + 1] ] = 2

∴ Range in cos–1 {log 1} and cos–1 {log 2} ∴ Rf is (π/2, cos–1 (log 2))

Ex.15 Find the range of the following functions


(i) f(x) = loge (sinx sinx + 1) where 0 < x < π/2.

π π
(ii) f(x) = loge (2 sin x + tan x – 3x + 1) where ≤x≤
6 3

Sol. (i) 0 < x < π/2 ⇒ 0 < sin x < 1


∴ Range of loge (sin x sin x + 1) for 0 < x < π/2 = Range of loge (xx + 1) for 0 < x < 1
x log e x
Let h(x) = xx + 1 = e +1

x loge x
∴ h′(x) = e (1 + loge x) ⇒ h′(x) > 0 for x > 1/e and h′(x) < 0 for x < 1/e
∴ h(x) has a minima at x = 1/e
Page # 12 FUNCTION

 ln x   1/ x 
lim   lim  
x →0  1/ x 
= 1 + e  −1/ x 
2
Also xlim = 1 + e0 = 2 and xlim
x →0

→0 +
h(x) = 1 + e →1−
h(x) = 2

1
 1 e
∴ 0 < x< 1 ⇒ 1 +   < (xx + 1) < 2
e

 1
 Y
  1 e 
1 +  
⇒ loge   (1, 2)
  e   < loge (x +1) < loge 2
x

  (0, 2)
f(x)
1 + (1/e)1/e
  −1
  O X
 log 1 + e e , log 2  1/e 1
∴ Range of f(x) =  e
  e

   

(ii) Let h(x) = (2 sin x + tan x – 3x +1) ⇒ h′(x) = (2 cos x + sec2 x – 3)

2 cos3 x − 3 cos2 x + 1
= ∴ h′(x) > 0 ⇒ 2 cos3 x – 3 cos2 x + 1 > 0
cos2 x

 1
(cos x – 1)2  cos x +  > 0 ∀ x ∈ [π/6, π/3] ⇒ h(x) is an increasing function of x
 2

 1 π
⇒ h(π/6) ≤ h(x) ≤ h(π/3) ⇒ loge  2 + −  ≤ log h(x) ≤ log (1 + 2 3 – π)
 3 2  e e

  1 π 
∴ Range of f(x) is log e  2 + − , log(1 + 2 3 − π)
  3 2 

(11) Equal or Identical Functions : Two functions f & g are said to be equal if :
(i) The domain of f = The domain of g ⇒ Df = Dg
(ii) The range of f = The range of g ⇒ Rf = Rg
(iii) f(x) = g(x), ∀ x ∈ their common domain.

1 x
Ex.16 Let f(x) = and g(x) = 2 then Df : R – {0} and Dg : R – {0}
x x

∴ Df = Dg ⇒ Hence both functions are identical

1
Ex.17 Let f(x) = sin x and g(x) = then Df : x ∈ R and Dg : x ∈ R – {nπ}
cos ec x
∴ Df ≠ Dg ⇒ Hence both functions are non-identical
FUNCTION Page # 13

D. CLASSIFICATION OF FUNCTIONS

(1) One − One Function (Injective mapping) : A function f : A → B is said to be a one−one


function or injective mapping if different elements of A have different f images in B. Thus for x1,
x2 ∈ A & f(x1) , f(x2) ∈ B , f(x1) = f(x2) ⇔ x1 = x2 or x1 ≠ x2 ⇔ f(x1) ≠ f(x2).

Diagrammatically an injective mapping can be shown as

Remark :
(i) Any function which is entirely increasing or decreasing in its domain, is one−one .
(ii) If any line parallel to x−axis cuts the graph of the function atmost at one point,
then the function is one−one.
(2) Many−One function : A function f : A → B is said to be a many one function if two or more
elements of A have the same f image in B . Thus f : A → B is many one if for
x1, x2 ∈ A , f(x1) = f(x2) but x1 ≠ x2 .
Diagrammatically a many one mapping can be shown as

Remark :
(i) A continuous function f(x) which has atleast one local maximum or local minimum, is
many−one. In other words, if a line parallel to x−axis cuts the graph of the function
atleast at two points, then f is many−one.
(ii) If a function is one−one, it cannot be many−one and vice versa.
(iii) If f and g both are one-one, then fog and gof would also be one-one (if they exist).

x 2 − 8 x + 18
Ex.18 Show that the function f(x) = is not one-one.
x 2 + 4 x + 30
Sol. Test for one-one function
A function is one-one if f(x1) = f(x2) ⇒ x1 = x2

x12 − 8 x1 + 18 x 22 − 8 x 2 + 18
Now f(x1) = f(x2) ⇒ 2 =
x1 + 4 x1 + 30 x 22 + 4 x 2 + 30

⇒ 12x12 x 2 − 12x1x 22 + 12x12 − 12x 22 − 312x1 + 312x 2 = 0


26 − x 2
⇒ (x1 – x2) {12x1x2 + 12(x1 + x2) – 312} = 0 ⇒ x1 = x2 or x1 =
1+ x2
Since f(x1) = f(x2) does not imply x1 = x2 alone, f(x) is not a one-one function.
Page # 14 FUNCTION

Ex.19 Let f be an injective function such that f(x) f(y) + 2 = f(x) + f(y) + f(xy) ∀ x, y ∈ R.
If f(4) = 65 and f(0) ≠ 2, then show that f(x) – 1= x3 ∀ x ∈ R.

Sol. Given that f(x) f(y) + 2 = f(x) + f(y) + f(xy) ....(i)


Putting x = y = 0 in equation (i), we get f(0) f(0) + 2 = f(0) + f(0) + f(0)
or (f(0))2 + 2 = 3f(0) (f(0) – 2) (f(0) – 1) = 0 or f(0) = 1 (∵ f(0) ≠ 2) ....(ii)
or
Again putting x = y = 1 in equation (i) and repeating the above steps, we get
(f(1) – 2) (f(1) – 1) = 0
But f(1) ≠ 1 as f(x) is injective. ∴ f(1) = 2 ....(iii)
Now putting y = 1/x in equation (i), we get

 1  1  1  1
f(x) f   + 2 = f(x) + f   + f(1) or f(x) f   + 2 = f(x) + f   + 2
x x x x

 1  1  1  1
or f(x) f   = f(x) + f   or f(x) f   – f(x) – f   – 1 + 1 = 0
x x x x

  1    1    1 
or f(x) f   − 1 − 1.f   − 1 = 1 or {f(x) – 1} f   − 1 = 1 ...(iv)
  
x  x   x 

 1  1
Let f(x) – 1 = g(x) ⇒ f  – 1 = g 
x x

 1
∴ from equation (iv), we get g(x) g   = 1 which is only possible when
x

g(x) = ± xn ∴ f(x) = ± xn + 1 or f(x) = ± xn + 1 or 65 = ± 4n + 1

or 4n = 64 = (4)3 ∴ n=3 ∴ f(x) = x3 + 1 or f(x) – 1 = x3 (neglecting negative sign)

(3) Onto-function (Surjective mapping) : If the function f : A → B is such that each element in
B (co−domain) is the f image of atleast one element in A, then we say that f is a function of A
'onto' B . Thus f : A → B is surjective iff ∀ b ∈ B, ∃ some a ∈ A such that f (a) = b.

Diagramatically surjective mapping can be shown as

Note that : if range ≡ co−domain, then f(x) is onto.


FUNCTION Page # 15

(4) Into function : If f : A → B is such that there exists atleast one element in co−domain which is
not the image of any element in domain, then f(x) is into.

Diagramatically into function can be shown as

Remark :
(i) If a function is onto, it cannot be into and vice versa .

(ii) If f and g are both onto, then gof or fog may or may not be onto.

Thus a function can be one of these four types :

(a) one−one onto (injective & surjective)

(b) one−one into (injective but not surjective)

(c) many−one onto (surjective but not injective)

(d) many−one into (neither surjective nor injective)

Remark :
(i) If f is both injective & surjective, then it is called a Bijective function. Bijective functions are
also named as invertible, non singular or biuniform functions.
(ii) If a set A contains n distinct elements then the number of different functions defined from A → A
is nn & out of it n ! are one-one.
(iii) The composite of two bijections is a bijection i.e. if f & g are two bijections such that gof is
defined, then gof is also a bijection.

Ex.20 A function is defined as , f : D → R f (x) = cot −1 (sgn x) + sin −1 (x − {x}) (where {x} denotes
the fractional part function) Find the largest domain and range of the function. State with reasons
whether the function is injective or not . Also draw the graph of the function.

 π π 3π 
Sol. D [-1 , 2) , R =  , ,  f is many one
4 2 4 
Page # 16 FUNCTION

Ex.21 Find the linear function(s) which map the interval [ 0 , 2 ] onto [ 1 , 4 ].
Sol. Let f (x) = a x + b

3
f (0) = 1 & f (2) = 4 ⇒ b = 1 & a =
2

3
or f (0) = 4 & f (2) = 1 ⇒ b = 4 & a = −
2

3x 3x
Ans. : f (x) = + 1 or f (x) = 4 −
2 2

Ex.22 (i) Find whether f(x) = x + cos x is one–one.


(ii) Identify whether the function f(x) = –x3 + 3x2 – 2x + 4 ; R → R is ONTO or INTO
(iii) f(x) = x2 – 2x + 3; [0, 3] → A. Find whether f(x) is injective or not. Also find the set A, if
f(x) is surjective.
Sol. (i) The domain of f(x) is R. f′(x) = 1 – sin x.
f(x)
∴ f′(x) ≥ 0 ∀ x ∈ complete domain
6
and equality holds at discrete points only
∴ f(x) is strictly increasing on R. Hence f(x) is one-one.
3
(ii) As codomain ≡ range, therefore given function is ONTO 2
(iii) f′(x) = 2(x – 1); 0 ≤ x ≤ 3
x
− ve ; 0 ≤ x < 1 0 1 3
∴ f′(x) = 
+ ve ; 1 < x < 3

∴ f(x) is a non monotonic continuous function. Hence it is not injective.


For f(x) to be surjective, A should be equal to its range. From graph,, range is [2, 6]
∴ A ≡ [2, 6]

Ex.23 If f and g be two linear functions from [–1, 1] onto [0, 2] and φ : R+ – {–1, 1} → R be defined by

f (x)   1 
φ(x) = , then show that φ( φ( x )) + φ φ 2   ≥ 2.
g( x )   

Sol. Let h be a linear function from [–1, 1] onto [0, 2].


Let h(x) = ax + b, then h′(x) = a
If a > 0, then h(x) is an increasing function & h(–1) = 0 and h(1) = 2 ⇒ –a + b = 0 and a + b = 2
⇒ a = 1 & b = 1. Hence h(x) = x + 1.
If a < 0, then h(x) is a decreasing function & h(–1) = 2 and h(1) = 0 ⇒ –a + b = 2 and a + b = 0
⇒ a = –1 & b = 1. Hence h(x) = 1 – x
Now according to the question f(x) = 1 + x & g(x) = 1 – x

f(x) 1 − x 1+ x
or f(x) = 1 – x & g(x) = 1 + x ∴ φ(x) = = or
g( x ) 1 + x 1− x
FUNCTION Page # 17

 1− x    1  1
Case-I : When φ(x) =   , x ≠ –1 ; φ φ   =
 1+ x    x  x

 1+ x    1 
Case-II : When φ(x) =  , x ≠ 1 φ φ   = – x.
 1− x    x 

2
1  1 
In both cases, |φ(f(x)) + φ(φ(1/x)) | = x + (where x > 0) =  x −  +2 ≥ 2
x
 x

E. FUNCTIONAL EQUATION

Functional Equation is an equation where the unknown is a function. On solving such an equation we
obtain one or more functions as solutions. If x, y are independent variables, then :
(i) f(xy) = f(x) + f(y) ⇒ f(x) = k ln x or f(x) = 0 .
(ii) f(xy) = f(x) . f(y) ⇒ f(x) = xn , n∈R
(iii) f(x + y) = f(x) . f(y) ⇒ f(x) = a , a > 0 kx

(iv) f(x + y) = f(x) + f(y) ⇒ f(x) = kx, where k is a constant.

Ex.24 If f(x + y + 1) = ( f ( x ) + f ( y ) )2 and f(0) = 1 ∀ x, y ∈ R. Determine f(x).

Sol. Given f(x + y + 1) = ( f ( x ) + f ( y ) )2

Putting x = y = 0; then f(1) = ( f (0 ) + f (0) )2 = (1 + 1)2 = 22

Again putting x = 0, y = 1 Then f(2) = ( f (0 ) + f (0) )2 = (1 + 2)2 = 32

and for x = 1, y = 1; f(3) = ( f (1) + f (1) )2 = (2 + 2)2 = 42 Similarly, f(x) = (x + 1)2

Ex.25 Let f : R – {2} → R function satisfying the following functional equation,

 2x + 29 
2f(x) + 3f   = 100x + 80, ∀ x ∈ R – {2}. Determine f(x).
 x−2 

3  2x + 29 
Sol. We have, f(x) = – f   + 50x + 40 ...(i)
2  x−2 

2x + 29
Replacing x by in the given functional equation we get,
x−2

  2x + 29  
 2  + 29 
 2x + 29  3  x−2   + 50 2x + 29 
f  = − f   + 40
 x−2  2  2x + 29   x−2 
  −2 
  x−2  
Page # 18 FUNCTION

 2x + 29  3  2x + 29 
⇒ f  = − f ( x ) + 50  + 40 ...(ii)
 x−2  2  x−2 

putting (ii) in (i), we get,

9  2x + 29  9  2x + 29 
f(x) = f(x) – 75   – 60 + 50x + 40 ⇒ f(x) – f(x) = 20 – 50 x + 75  
4  x−2  4  x−2 

5  2x + 29  (2 x + 29 )
⇒ f(x) = 20 – 50x + 75   ⇒ f(x) = 16 – 40x + 60
4  x−2  ( x − 2)

Ex.26 Let f be a function from the set of positive integers to the set of real numbers i.e., f : N → R such that
(i) f(1) = 1; (ii) f(1) + 2f(2) + 3f(3) + ... + nf(n) = n (n + 1) f(n) for n ≥ 2 then find the value of f (1994).
Sol. Given f(1) + 2f (2) + 3f(3) + ... + nf(n) = n(n + 1) f(n) ...(1)

Replacing n by (n + 1) then

f(1) + 2f(2) + 3f(3) + .... + nf(n) = n (n + 1) f(n + 1) = (n +1) (n + 2) f(n + 1) ...(2)

Subtracting (1) from (2) then we get

(n + 1) f(n + 1) = (n + 1) (n + 2) f (n + 1) – n (n + 1) f(n) ⇒ nf(n) = (n + 1) f(n + 1)

From which we conclude that 2f(2) = 3f (3) = 4f (4) = ... = nf(n)

Substituting the value of 2f(2), 3f(3), .... in terms of nf(n) in (1), we have
f (1) 1
f(1) + (n – 1) nf (n) = n(n + 1) f(n) ⇒ f(1) = 2n f(n) ∴ f(n) = = (∵ f(1) = 1)
2n 2n
1 1
⇒ f(1994) = =
2.1994 3988

Ex.27 Let f be a function satisfying 2f(xy) = {f(x)}y + {f(y)}x and f(1) = k ≠ 1. Prove that (k – 1) ∑ f (r ) = k
r =1
n+1
–k

Sol. Given that 2f(xy) = f(x))y + (f(y))x ...(1)


replacing y by 1, we get 2f(x) = f(x) + (f(1))x or f(x) = f(1)x = kx {∵ f(1) = k} ....(2)

∴ ∑ f (r ) = f(1) + f(2) + .............+f(2) = k + k


r =1
1 2
+ k3 + ....... + kn

k(k n − 1) k n+1 − k
n

=
(k − 1)
=
(k − 1)
or (k – 1) ∑ f (r ) = (k
r =1
n+1
– k)

Ex.28 Determine all the functions f : R → R such that f(x) f(y) – f(xy) = x + y for all x, y ∈ R ...(1)
Sol. Put x = y = 0 in (1). Hence f(0) f(0) – f(0) = 0. This implies that f(0) = 0 or f(0) = 1. If f(0) = 0 then f(x)
f(0) – f(0) = x + 0. Hence, x = 0 for all x ∈ R, a contradiction. Hence, f(0) = 1.
Substituting y = 0, in (1) we get f(x) . f(0) – f(0) = x + 0 ⇒ f(x) = x + 1 is the only solution of (1).
FUNCTION Page # 19

F. COMPOSITE FUNCTIONS

Let f: X → Y1 and g: Y2 → Z be two functions and the set D = {x ∈ X: f(x) ∈ Y2}. If D ≡/ φ , then the
function h defined on D by h(x) = g{f(x)} is called composite function of g and f and is denoted by gof.
It is also called function of a function.
Remark : Domain of gof is D which is a subset of X (the domain of f). Range of gof is a subset of the
range of g. If D = X, then f(X) ⊆ Y2.
Properties of composite functions :
(i) The composite of functions is not commutative i.e. gof ≠ fog.
(ii) The composite of functions is associative i.e. if f, g, h are three functions such that fo (goh) &
(fog) oh are defined, then fo (goh) = (fog) oh.

 π π
Ex.29 Let f(x) = ex ; R+ → R and g(x) = sin–1 x; [–1, 1] →  − ,  . Find domain and range of fog (x)
 2 2

 π π
Sol. Domain of f(x) : (0, ∞), Range of g(x) :  − , 
 2 2
 π
The values in range of g(x) which are accepted by f(x) are  0, 
2  
π π
⇒ 0 < g(x) ≤ 0 < sin–1 x ≤ 0<x ≤1
2 2
Hence domain of fog(x) is x ∈ (0, 1]
g f
(0, 1] (0, π/2]
0 π/2
(e , e ]
Range
Domain
–1
sin x ex
Therefore Domain : (0, 1], Range : (1/ eπ/2]

[| f (| x |) |], x ∈ ( −1, 0) ∪ (0, 1)


Ex.30 Let f(x) = x2 – 1 and g(x) =  . Then find the range of n ([|g(x)|])
1, otherwise

(where [ * ] denotes the greatest integer function)


Sol. | f ( |x| ) | = 1 – x2, x ∈ (–1, 0) ∪ (0, 1) ⇒ [ | f ( |x| ) | ] = 0, x ∈ (–1, 0) ∪ (0, 1)
Also [f(x)] = –1, x ∈ (–1, 0) ∪ (0, 1) ⇒ | [ f(x) ] | = 1, x ∈ (–1, 0) ∪ (0, 1)
∴ g(x) = 1, x ∈ R ⇒ range of n ( [ | g(x) | ] ) = {0}.

x −1 2
Ex.31 Let f(x) = , f (x) = f{f(x)}, f3 (x) = f{f2(x)},.....fk + 1 (x) = f{fk(x)}. for k = 1, 2, 3,...., Find f1998 (x).
x +1

x −1 −1
−1 −1
− −1 f 2 (x) − 1 x +1
= x +1
x −1 f 1 x
= = =
Sol. f(x) = , f (x) = f{f(x)} = f + 1 x − 1
2
x , f (x) = f{f(x)} = f 2 ( x ) + 1 − 1
3
x −1,
x +1 +1 +1
x +1 x
Page # 20 FUNCTION

x +1
−1
f 3 (x) − 1 x − 1 x +1
=
f4 = f{f3(x)} = f 3 ( x ) + 1 x + 1 = x, f5(x) = f{f4 (x)} = = f(x)
+1 x −1
x −1

Thus, we can see that fk(x) repeats itself at intervals of k = 4.

−1
Hence, we have f1998(x) = f2(x) = [∴ 1998 = 499 × 4 + 2]
x

Ex.32 Let g : R → R be given by g(x) = 3 + 4x. If gn(x) = gogo....og(x), show that fn(x) = (4n – 1) + 4nx if
g–n (x) denotes the inverse of gn (x).
Sol. Since g(x) = 3 + 4x
∴ g2(x) = (gog) (x) = g {g (x)} = g (3 + 4x) = 3 + 4 (3 + 4x) or g2(x) = 15 + 42x = (42 – 1) + 42x
Now g3(x) = (gogog) x = g {g2 (x) } = g (15 + 42 x) = 3 + 4 (15 + 42 x) = 63 + 43 x = (43 –1) + 43x
Similarly we get gn(x) = (4n – 1) + 4nx
Now leg gn (x) = y ⇒ x = g–n(y) ...(1)
∴ y = (4 – 1) + 4 x or
n n n
x = (y + 1 – 4 )4 –n
...(2)
From (1) and (2) we get g–n (y) = (y + 1 – 4n) 4–n. Hence g–n (x) = (x + 1 – 4n) 4–n

Ex.33 If f(x) = | |x – 3| – 2 | ; 0 ≤ x ≤ 4 and g(x) = 4 – |2 – x| ; –1 ≤ x ≤ 3 then find fog(x).

1 − g( x ) 0 ≤ g( x ) < 1 1 − g( x ) for no value


2 + x − 1 ≤ x < 2  
− ≤ < g( x ) − 1 − 1≤ x < 1
∴ fog(x) =  = 
g( x ) 1 1 g( x ) 3
Sol. g(x) = 
6 − x 2 ≤ x ≤ 3 5 − g( x ) 3 ≤ g( x ) ≤ 4

5 − g( x )
 1≤ x ≤ 3

g(x)

 2 + x − 1 − 1≤ x < 1 x + 1 − 1≤ x < 1 2
 
5 − (2 + x ) 1 ≤ x < 2 3 − x 1≤ x < 2
=  =  1
5 − ( 6 − x ) 2 ≤ x ≤ 3 x −1 2 ≤ x ≤ 3
  x
–1 1 2 3

Ex.34 Prove that f(n) = 1 – n is the only integer valued function defined on integers such that
(i) f(f(n)) = n for all n ∈ Z and (ii) f(f(n + 2) + 2) = n for all n ∈ Z and (iii) f(0) = 1.
Sol. The function f(n) = 1 – n clearly satisfies conditions (i), (ii) and (iii). Conversely, suppose a function
f : Z → Z satisfies (i), (ii) and (iii). Applying f to (ii) we get, f(f(f(n + 2) + 2) ) ) = f(n)
and this gives because of (i), f(n + 2) + 2 = f(n), ........(1)
for all n ∈ Z. Now using (1) it is easy to prove by induction on n that for all n ∈ Z,

 f (0) − n if n is even
f(n) = 
f (1) + 1 − n if n is odd
Also by (iii), f(0) = 1. Hence by (i), f(1) = 0. Hence f(n) = 1 – n for all n ∈ Z.
FUNCTION Page # 21

G. GENERAL DEFINITION

(1) Identity function : A function f : A → A defined by f(x) = x ∀ x ∈ A is called the identity of A &
denoted by IA. Ex : f : R+ → R+ ; f(x) = en x and f : R → R ; f(x) = n ex
Every Identity function is a bijection.
(2) Constant function : A function f : A → B is said to be constant function. If every element of set
A has the same functional image in set B i.e. f : A → B ; f(x) = c ∀ x ∈ A & c ∈ B is called constant
function.
(3) Homogeneous function : A function is said to be homogeneous w.r.t. any set of variables when
each of its term is of the same degree w.r.t. those variables.
(4) Bounded Function : A function y = f(x) is said to be bounded if it can be express is the form of
a ≤ f(x) ≤ b where a and b are finite quantities.
Ex : –1 ≤ sin x ≤ 1 ; 0 ≤ {x} < 1 ; –1 ≤ sgn (x) ≤ 1 but ex is not bounded.
Ex : Any function having singleton range like constant function.
(5) Implicit function & Explicit function : If y has been expressed entirely in terms of ‘x’ then it is
called an explicit function.
If x & y are written together in the form of an equation then it is known as implicit equation
corresponding to each implicit equation there can be one, two or more explicit function satisfying it
Ex : y = x3 + 4x2 + 5x → Explicit function
Ex : x + y = 1 → Implicit equation
Ex : y = 1 – x → Explicit function

H. EVEN & ODD FUNCTIONS

Function must be defined in symmetric interval [–x, x]


If f (−x) = f (x) for all x in the domain of ‘f’ then f is said to be an even function.
e.g. f (x) = cos x ; g (x) = x² + 3.
If f (−x) = −f (x) for all x in the domain of ‘f’ then f is said to be an odd function.
e.g. f (x) = sin x ; g (x) = x3 + x.

Remark :
(a) f (x) − f (−x) = 0 ⇒ f (x) is even & f (x) + f (−x) = 0 ⇒ f (x) is odd .
(b) A function may be neither even nor odd.
(c) Inverse of an even function is not defined.
(d) Every even function is symmetric about the y−axis & every odd function is symmetric about the
origin .
(e) A function (whose domain is symmetric about origin) can be expressed as a sum of an even & an
f ( x )+f ( − x ) f ( x )−f ( − x )
odd function. e.g. f ( x ) = +
2 2
EVEN ODD

(f) The only function which is defined on the entire number line & is even and odd at the same time is
f(x) = 0 .

(g) If f and g both are even or both are odd then the function f.g will be even but if any one of them
is odd and other even then f.g will be odd.
Page # 22 FUNCTION

Ex.35 Which of the following functions is odd ?


(A) sgn x + x2000 (B) | x | – tan x (C) x3 cot x (D) cosec x55
Sol. Let’s name the function of the parts (A), (B), (C) and (D) as f(x), g(x), h(x) & φ(x) respectively. Now
(A) f(–x) = sgn (–x) + (–x)2000 = –sgn x + x2000 ≠ f(x) & ≠ –f(x) ∴ f is neither even nor odd.
(B) g(–x) = |–x| – tan (–x) = |x| + tan x ∴ g is neither even nor odd.
3
(C) h(–x) = (–x) cot (–x) 3 3
= –x (–cot x) = x cot x = h(x) ∴ h is an even function
(D) φ(–x) = cosec (–x)55 = cosec (–x55) = –cosec x55 = – φ(x) ∴ φ is an odd function.

Alternatively
(A) f(x) = sgn (x) + x2000 = O + E = neither E nor O
(B) g(x) = E – O = Neither E nor O
(C) h(x) = O × O = E
(D) f(–x) = O o O = O ∴ (D) is the correct option

3
Ex.36 f(x) = (tan x5) e x sgn x 7 is

(A) an even function (B) an odd function


(C) neither even nor odd function (D) none of these
3
Sol. f(x) = (tan (x5)) e x sgn ( x 7 )

↓ ↓ ↓ ↓ ↓
O (O) e O × O (O)
= O × eO × O = O × eE =O×E=O

 π
x tan x, 0 < x ≤ 2
Ex.37 Let f: [–2, 2] → R be a function if f(x) =  Define f for x ∈ [–2, 0] so that
π π
 [ x ], x≤2
 2 2

(i) f is an odd function (ii) f is an even function (where [*] denotes the greatest integer function)

 π  π  π
x tan x, 0 < x ≤ 2 ( − x ) tan( − x ), 0 < x ≤ 2 x tan x, − 2 ≤ x < 0
Sol. Since f(x) =  π π ∴ f(–x) =  π or f (–x) = 
 [ x], π π π
x≤2  [ − x ], < −x ≤ 2  [ − x ], − 2 ≤ x < −
 2 2  2 2  2 2

 π
− x tan x, − 2 ≤ x < 0
(i) If f is an odd function then f(x) = –f (–x) =
 π π
 − [ − x ], − 2 ≤ x < −
 2 2
 π
x tan x, − 2 ≤ x < 0
(ii) If f is an even function ∴ f(x) = f(–x) = π
 π
 [ − x], − 2 ≤ x < −
2 2
FUNCTION Page # 23

Ex.38 Let f(x) = ex + sin x be defined on the interval [–4, 0]. Find the odd and even extension of f(x) in the
interval [–4, 4].
Sol. Odd Extension : Let g0 be the odd extension of f(x), then

 f (x) ; x ∈ [ −4,0]  e x + sin x ; x ∈ [ −4,0]


g0 (x) =  =  −x
− f ( − x ) ; x ∈ [0,4] − e + sin x ; x ∈ [0,4]
Even Extension : Let ge be the even extension of f(x), then
 f ( x ) ; x ∈ [ −4,0]  e x + sin x ; x ∈ [ −4,0]
ge (x) =  =  −x
f ( − x ) ; x ∈ [0,4] e − sin x ; x ∈ [0,4]

I. PERIODIC FUNCTION

A function f(x) is called periodic if there exists a positive number T (T > 0) called the period of the
function such that f (x + T) = f(x), for all values of x and x + T within the domain of f(x). The least
positive period is called the principal or fundamental period of f.
e.g. The function sin x & cos x both are periodic over 2π & tan x is periodic over π.
Remark :
(a) A constant function is always periodic, with no fundamental period.
1
(b) If f(x) has a period p, then and f (x) also has a period p.
f( x )
(c) if f(x) has a period T then f(ax + b) has a period T/a (a > 0).
f ( x)
(d) If f(x) has a period T1 & g(x) also has a period T2 then period of f(x) ± g(x) or
g( x ) is L.C.M of
T1 & T2 provided their L.C.M. exists. However that L.C.M. (if exists) need not to be fundamental
f( x)
period. If L.C.M. does not exists then f(x) ± g(x) or f(x) . g(x) or
g( x ) is nonperiodic e.g. |sin x|
has the period π, |cos x| also has the period π
∴ |sin x| + |cos x| also has a period π. But the fundamental period of |sin x| + |cos x| is π/2.
(e) If g is a function such that gof is defined on the domain of f and f is periodic with T, then gof is
also periodic with T as one of its periods. Further if
# g is one-one, then T is the period of gof
# g is also periodic with T’ as the period and the range of f is a subset of [0, T’], then T is the
period of gof
(f) Inverse of a periodic function does not exist.

Ex.39 Find period of the following functions

x x
(i) f(x) = sin + cos (ii) f(x) = {x} + sin x
2 3

3x x 2x
(iii) f(x) = cos x . cos 3x (iv) f(x) = sin – cos – tan .
2 3 3
Sol. (i) Period of sin x/2 is 4π while period of cos x/3 is 6π. Hence period of sin x/2 + cos x/3 is 12 π
{L.C.M. or 4 & 6 is 12}
(ii) Period of sin x = 2π ; Period of {x} = 1; but L.C.M. of 2π & 1 is not possible ∴ it is aperiodic
Page # 24 FUNCTION

 2π 
(iii) f(x) = cos x . cos 3x ; Period of f(x) is L.C.M. of  2π,  = 2π
 3 

but 2π may or may not be the fundamental period. The fundamental period can be , where
n
n ∈ N. Hence cross-checking for n = 1, 2, 3..... we find π to be fundamental period
f(π + x) = (–cos x) (– cos 3x) = f(x)

2π 2π π 4π 2π
(iv) Period of f(x) is L.C.M. of , , = L.C.M. of , 6π, = 12π
3 / 2 1/ 3 3 / 2 3 3

Ex.40 Prove that sin (1/x), (x ≠ 0) is a non-periodic function


Sol. Let f(x) = sin (1/x) be periodic with period T, T ≠ 0, T > 0 ∴ f(x + T) = f(x)

 1   1 1 1
⇒ sin   = sin  ⇒ = nπ + (–1)n ...(1)
 x + T  x x+T x

1 1
Put x = T and x = 2T in (1), then = nπ + (–1)n ...(2)
2T T

1 1
and = np + (–1)n ...(3)
3T 2T

1 1 1
Subtracting (3) from (2), we get = ( −1)n . or = (–1)n which is impossible.
6T 2T 3
Hence sin (1/x), (x ≠ 0) is non-periodic function.

Ex.41 If f(x) = sin x + cos ax is a periodic function, show that a is a rational number.
Sol. Given f(x) = sin x + cos ax
2π 2π
∵ Period of sin x = and period of cos ax =
1 a
 2 π 2π  L.C.M. of {2π, 2π} 2π
Hence period of f(x) = L.C.M. or  ,  = =
1 a H.C.F. of {1, a} k
where k = H.C.F. of 1 and a
1 a
∴ = integer = q (say), (≠0) and = integer = p (say)
k k
a/k p p
∴ = ⇒ a= ∴ a = rational number
1/ k q q

Ex.42 Given below is a partial graph of an even periodic function


f whose period is 8. If [*] denotes greatest integer function
then find the value of the expression.

  7 
f (–3) + 2 | f (–1) | +  f   + f (0) + arc cos (f(–2)) + f (–7) + f (20)
  8 
FUNCTION Page # 25

Sol. f (–3) = f (3) = 2 [ f (x) is an even function, ∴ f (– x) = f (x) ]

again f (–1) = f (1) = – 3

∴ 2 | f (–1) | = 2 | f (1) | = 2 | – 3 | = 6

7   7 
from the graph, –3< f  <–2 ∴ f   = – 3
8   8 

f (0) = 0 (obviously from the graph)

cos–1 (f (−2)) = cos–1 (f (2) ) = cos–1(1) = 0

f (–7) = f (– 7 + 8) = f (1) = – 3 [f (x) has period 8]


f (20) = f (4 + 16) = f (4) = 3 [ f (nT + x) = f (x) ]
sum = 2 + 6 – 3 + 0 + 0 – 3 + 3 = 5

Ex.43 Check whether the function defined by f(x + λ) = 1 + 2f ( x ) − f 2 ( x ) ∀ x ∈ R, is periodic or not, if


periodic, then find its period.
Sol. The given function is true if 2f(x) – f2(x) ≥ 0 ⇒ f(x)[f(x) – 2] ≤ 0 ⇒ 0 ≤ f(x) ≤ 2 ....(i)
Also from the given function, it is clear that f(x + λ) ≥ 1 ⇒ f(x) ≥ 1 ....(ii)
From (i) and (ii), we conclude that 1 ≤ f(x) ≤ 2
Again, we have {f(x + λ) – 1}2 = 2f(x) – f2(x) ⇒ {f(x + λ) – 1}2 = – {(f(x) – 1)2} + 1 ...(iii)
Replacing x by x + λ in above equation, we get {f(x + 2λ) – 1}2 = – {f(x + λ) – 1}2 + 1 ...(iv)
From (iv) – (iii), we get
{f(x + 2λ) – 1}2 = {f(x) – 1)}2 ⇒ f(x + 2λ) = f(x) ⇒ f is periodic function with period 2λ.

Ex.44 If the periodic function f(x) satisfies the equation f(x + 1) + f(x –1) = 3 f(x) ∀ x ∈ R then find the
period of f(x)

Sol. We have f(x + 1) + f(x – 1) = 3 f(x) ∀ x ∈ R ...(1)

Replacing x by x – 1 and x + 1 in (1) then f(x) + f(x – 2) = 3 f(x – 1) ...(2)

and f(x + 2) + f(x) = 3 f(x + 1) ...(3)

Adding (2) and (3), we get 2f(x) + f(x – 2) + f(x + 2) = 3 (f (x – 1) + f(x + 1))

⇒ 2f (x) + f(x – 2) + f(x + 2) = 3 . 3 f(x) [From (1)]

∴ f(x + 2) + f(x – 2) = f(x) ...(4)


Replacing x by x + 2 in equation (4) then f (x + 4) + f (x) = f (x + 2) ...(5)
Adding equations (4) and (5), we get f(x + 4) + f (x – 2) = 0 ...(6)
Again replacing x by x + 6 in (6) then f (x + 10) + f (x + 4) = 0 ...(7)
Subtracting (6) from (7), we get f (x + 10) – f (x – 2) = 0 ...(8)
Replacing x by x + 2 in (8) then f (x + 12) – f(x) = 0 or f (x + 12) = f(x)
Hence f(x) is periodic function with period 12.
Page # 26 FUNCTION

Ex.45 Let f (x, y) be a periodic function satisfying f(x, y) = f(2x + 2y, 2y – 2x) for all x, y; Define g(x) = f(2x , 0).
Show that g(x) is a periodic function with period 12.
Sol. Since f(x, y) = f(2x + 2y, 2y – 2x) ...(1)
∴ f(2x + 2y, 2y – 2x)
= f(2(2x + 2y) + 2(2y – 2x), 2(2y – 2x) – 2(2x + 2y)) = f(8y, –8x) ...(2)
From (1) and (2) we get f(x, y) = f(8y, – 8x) ...(3)
∴ f(8y, – 8x) = f(8 (–8x), –8 (8y)) = f(–64, – 64y) ...(4)
From (3) and (4) we get f(x, y) = f(–64x, –64y) ...(5)
∴ f(–64x, – 64y) = f(–64 (–64x), –64(–64y)) = f(2 x, 2 y) 12 12
...(6)
From (5) and (6) we get f(x, y) = f(2 x, 2 y) ⇒
12 12 12
f(x, 0) = f(2 x, 0)
Replace x by 2 thex x
f(2 , 0) = f(2x + 12
, 0) ⇒ g(x) = g(x + 12) {∵ g(x) = f(2x, 0)}
Hence g(x) is periodic with period 12.

J. INVERSE OF A FUNCTION

Let f : A → B be a one−one & onto function, then their exists a unique function
g : B → A such that f(x) = y ⇔ g(y) = x, ∀ x ∈ A & y ∈ B . Then g is said to be inverse of f. Thus
g = f−1 : B → A = {(f(x), x)  (x, f(x)) ∈ f}.
Properties of inverse function :
(i) The inverse of a bijection is unique, and it is also a bijection.
(ii) If f : A → B is a bijection & g : B → A is the inverse of f, then fog = IB and
gof = IA , where IA & IB are identity functions on the sets A & B respectively.
(iii) The graphs of f & g are the mirror images of each other in the line y = x.
(iv) Normally points of intersection of f and f–1 lie on the straight line y =x. However it must be noted
that f(x) and f–1(x) may intersect otherwise also.
(v) In general fog(x) and gof(x) are not equal. But if either f and g are inverse of each other or
atleast one of f, g is an identity function, then gof = fog.
(vi) If f & g are two bijections f : A → B , g : B → C then the inverse of gof exists
and (gof)−1 = f−1 o g−1.

Ex.46 Find the inverse of the function f(x) = ln(x2 + 3x +1); x ∈ [1, 3] and assuming it to be an onto function.

2x + 3
Sol. Given f(x) = ln (x2 + 3x + 1) ∴ f′(x) = > 0 ∀ x ∈ [1, 3]
( x + 3 x + 1)
2

which is a strictly increasing function. Thus f(x) is injective, given that f(x) is onto. Hence the given
function f(x) is invertible. Now let y = f(x) = ln (x2 + 3x + 1) then x = f–1 (y) ...(1)
2
and y = ln (x + 3x + 1) ⇒ y
e = x + 3x + 12
⇒ x + 3x + 1 – ey = 0
2

− 3 ± 9 − 4.1.(1 − e y ) − 3 ± (5 + 4e y ) − 3 + ( 5 + 4e y )
∴ x= = = (∴ x ∈ [1, 3]) ...(2)
2 2 2
− 3 + ( 5 + 4e y ) − 3 + ( 5 + 4e x )
From (1) and (2), we get f–1 (y) = Hence f–1(x) =
2 2
FUNCTION Page # 27

 x, x <1
 2
x , 1≤ x ≤ 4
Ex.47 Find the inverse of the function f(x) = 
8 x , x>4

 x, x <1
 2
Sol. Given f(x) = f(x) =  x , 1 ≤ x ≤ 4
8 x , x>4

 y, y <1  y, y <1
 
Let f(x) = y ⇒ x = f–1(y) ....(1) ∴x=  y, 1 ≤ y ≤ 4 =  y, 1 ≤ y ≤ 16
y 2 / 64, y 2 / 64 > 4 y 2 / 64, y > 16
 

 y, y <1  x, x <1
 
f–1(y) =  y, 1 ≤ y ≤ 16 [From (1)]. Hence f–1 (x) =  x , 1 ≤ x ≤ 16 .
 y 2 / 64, y > 16  2
x > 16
 y / 64,

[
3,∞
Ex.48 A function f : 2 ) → [ 74 , ∞) defined as, f (x) = x2 − 3 x + 4 . Then compute f −1 (x) and find the
solution of the equation, f (x) = f −1 (x) .
Sol. f (x) = y = x2 − 3 x + 4 ⇒ x2 − 3 x + (4 − y) = 0

3 ± 9 − 4 ( 4 − y) 3+ 4y − 7 4x − 7
x = = f − 1 (y) = 3 +
2 2 2
graphs of f − 1 (x) & f (x) intersect each other at y = x ⇒ f (x) = x ⇒ x2 − 3 x + y = x ⇒ x = 2

Ex.49 Let l1 be the line 4x + 3y = 3 and l2 be the line y = 8x. L1 is the line formed by reflecting l1 across
the line y = x and L2 is the line formed by reflecting l2 across the x-axis. If θ is the acute angle
between L1 and L2 such that tan θ = a/b, where a and b are coprime then find (a + b).
3 − 4x
Sol. l1 : 4x + 3y = 3 f (x) = y = ....(1)
3
since f (x) and f–1(x) are the mirror images of each other in the line y = x hence we find f–1(x).
now y = f (x) ⇒ f –1(y) =x
3(1 − y ) 3(1 − y ) 3(1 − x )
from (1) x= ; f–1(y) = ∴ f–1(x) =
4 4 4
4y = 3 – 3x L1 = 3x + 4y – 3 = 0
m1 = – 3/4 |||ly L2 = y = – 8x with m2 = – 8

3
m 2 − m1 −8+ − 29 29
if θ is the acute angle between the lines tan θ = = 4 = ⇒
1 + m1m 2  3 28 28
1 + ( −8) − 
 4

⇒ a = 29 and b = 28 ∴ a + b = 29 + 28 = 57
Page # 28 FUNCTION

EXERCISE – I SINGLE CORRECT (OBJECTIVE QUESTIONS)

− log0.3 ( x − 1) Sol.
1. The domain of the function f(x)= is
x 2 + 2x + 8
(A) (1, 4) (B) (–2, 4) (C) (2, 4) (D) [2, ∞)
Sol.

2. The domain of the function

  1  
f(x) = log1/2  − log2  1 +  − 1 is 4. If domain of f(x) is (– ∝, 0] then domain of
 4
x  
  2
f(6{x} – 5{x} + 1) is
(A) 0 < x < 1 (B) 0 < x ≤ 1 (C) x ≥ 1 (D) null set
(where {*} represetns fractional part function)
Sol.
 1 1
(A) ∪ n + 3 ,n + 2  (B) (–∞, 0)
n∈Ι

 1 
(C) ∪ n + 6 n + 1
n∈Ι
(D) None of these

Sol.

2
3. If q – 4 p r = 0, p > 0, then the domain of the
3 2
function, f(x) = log (px + (p + q) x + (q + r) x + r) is
5. Find domain of the function
 q   q 
(A) R – −  (B) R – ( −∞,−1] ∪ − 
 2p    2p   2x − 1 
f(x) = − log x + 4  log2 
2 
3+x 
  q 
(C) R– ( −∞,−1] ∩ −  (D) none of these (A) (–4, –3) ∪ (4, ∞) (B) (– ∞, –3) ∪ (4, ∞)
  2p  (C) (– ∞, – 4) ∪ (3, ∞) (D) None of these
FUNCTION Page # 29

Sol.
6. The domain of the function log1/ 3 log4 ([ x ]2 − 5) is
(where [x] denotes greatest integer function)
(A) [–3, –2) ∪ [3, 4) (B) [–3, –2) ∪ (2, 3]
(C) R – [–2, 3) (D) R – [–3, 3]
Sol.

7. Let f(x) = (x12 – x9 + x4 – x + 1)–1/2. the domain of


the function is
(A) (1, + ∞) (B) (–∞, –1) (C) (–1, 1) (D) (–∞, ∞)
Sol.

8. Range of f(x) = 4 x + 2x + 1 is
(A) (0, ∞) (B) (1, ∞) (C) (2, ∞) (D) (3, ∞)
Sol.
Page # 30 FUNCTION

9. Range of f(x) = log 5 { 2 (sin x –cos x) + 3} is

 3
(A) [0, 1] (B) [0, 2] (C) 0,  (D) None of these
2  
Sol.

10. The range of the functin


2
f(x) = log 2 ( 2– log2 (16 sin x + 1)) is
(A) (–∞, 1) (B) (–∞, 2) (C) (–∞, 1] (D) (–∞, 2]
Sol.

12. The range of the function f(x) = 7 – xPx – 3 is


(A) {1, 2, 3} (B) {1, 2, 3, 4, 5, 6}
(C) {1, 2, 3, 4} (D) {1, 2, 3, 4, 5}
Sol.
11. If [2 cos x] + [sin x] = –3, then the range of the

function, f(x) = sin x + 3 cos x in [0, 2π] is


(where [ * ] dentoes greatest integer function)
(A) [–2, –1) (B) (–2, –1]

(C) (–2, –1) (D) [–2, – 3)


Sol.
FUNCTION Page # 31

15. Let f be a real valued function defined by


x e x − e −|x|
cos 1 1 f(x) = then the range of f(x) is
2 e x + e|x|
x x
13. Range of the function f(x)= 1 cos − cos is
2 2  1
x (A) R (B) [0, 1] (C) [0, 1) (D) 0, 
− cos 1 −1  2
2
Sol.
(A) [0, 2] (B) [0, 4] (C) [2, 4] (D) [1, 3]
Sol.

14. In the square ABCD with side AB = 2, two points


M & N are on the adjacent sides of the square such
that MN is parallel to the diagonal BD. If x is the
distance of MN from the vertex A and sin2 x + 4 sin x + 5
16. If f(x) = , then range of f(x) is
f(x) = Area (∆AMN), then range of f(x) is 2 sin2 x + 8 sin x + 8

(A) (0, 2 ] (B) (0, 2] (C) (0,2 2 ] (D) (0, 2 3 ]


1  5  5  5 
Sol. (A)  , ∞  (B)  ,1 (C)  ,1 (D)  , ∞ 
2  9  9  9 
Sol.
Page # 32 FUNCTION

19. The sum

 1  1 1  1 2  1 3   1 1999 
 2  +  2 + 2000  +  2 + 2000  +  2 + 2000  + ...... +  2 + 2000 
         
is equal to
(where [ * ] denotes the greatest integer function)
(A) 1000 (B) 999 (C) 1001 (D) None of these
17. The number of solution(s) of the equation Sol.
[x] + 2{–x} = 3x, is/are
(where [ * ] represents the greatest integer function
and { * } denotes the fractional part of x)
(A) 1 (B) 2 (C) 3 (D) 0
Sol.

20. Which of the following represents the graph of


f(x) = sgn ([x + 1])

1 1
–1 –1
(A) (B)
–1 –1

1 1
–1 –1
(C) (D)
1 1
–1 –1

Sol.
18. The number of solutions of the equation
[sin–1 x] = x – [x] is
(where [ * ] denotes the greatest integer function)
(A) 0 (B) 1 (C) 2 (D) infinitely many
Sol.

21. If f(x)=2 sin2θ+4 cos (x+θ) sin x. sin θ+cos (2x+2θ)

π 
then value of f2(x) + f2  − x  is
4 
(A) 0 (B) 1 (C) –1 (D) x2
FUNCTION Page # 33

Sol. Sol.

25. The function f : [2, ∞) → Y defined by


2
f(x) = x – 4x + 5 is both one–one & onto if
(A) Y = R (B) Y = [1, ∞) (C) Y = [4, ∞) (D) Y = [5, ∞)
22. If A, B, C are three decimal numbers and Sol.
p = [A + B + C] and q = [A] + [B] + [C] then maximum
value of p – q is (where [ * ] represents greatest
integer function).
(A) 0 (B) 1 (C) 2 (D) 3
Sol.

26. Let f : R → R be a function defined by

2x 2 − x + 5
f(x) = then f is
7 x 2 + 2x + 10
23. Let f(x) = ax2 + bx + c, where a, b, c are rational (A) one – one but not onto
and f : Z → Z, where Z is the set of integers. Then (B) onto but not one – one
a + b is (C) onto as well as one – one
(A) a negative integer (B) an integer (D) neither onto nor one – one
(C) non-integral rational number (D) None of these Sol.
Sol.

27. Let f : R → R be a function defined by


3 2
24. Which one of the following pair of functions are f(x) = x + x + 3x + sin x. Then f is
identical ? (A) one – one & onto (B) one – one & into
(ln x)/2 (C) many one & onto (D) many one & into
(A) e and x Sol.
–1 –1
(B) tan (tan x) & cot (cot x)
2 4 2 4
(C) cos x + sin x and sin x + cos x

|x|
(D) and sgn (x) where sgn(x) stands for signum
x
function.
Page # 34 FUNCTION

30. If f(x) = 2[x] + cos x, then f: R → R is


(where [ * ] denotes greatest integer function)
(A) one–one and onto (B) one–one and into
(C) many–one and into (D) many–one and onto
Sol.

4a − 7 3 2
28. If f(x) = x + (a – 3) x + x + 5 is a one–
3
one function, then
(A) 2 ≤ a ≤ 8 (B) 1 ≤ a ≤ 2
(C) 0 ≤ a ≤ 1 (D) None of these
Sol.

31. If f : R → S, defined by f(x) = sin x – 3 cosx + 1,


29. Let f: (e, ∞) → R be defined by f(x) = ln (ln(ln x)),
then is onto, then the interval of S is
(A) f is one one but not onto (A) [0, 3] (B) [–1, 1] (C) [0, 1] (D) [–1, 3]
(B) f is onto but not one – one Sol.
(C) f is one–one and onto
(D) f is neither one–one nor onto
Sol.

32. The function f : R → R defined by f(x) = 6x + 6|x| is


(A) one-one and onto (B) many-one and onto
(C) one-one and into (D) many-one and into
Sol.
FUNCTION Page # 35

33. If the real-valued function f(x) = px + sinx is a 36. Function f : (– ∞, 1) → (0, e5] defined by
bijective function, then the set of all possible values −( x 2 −3 x + 2 )
of p ∈ R is f(x) = e is
(A) R – {0} (B) R (C) (0, ∞) (D) None of these (A) many one and onto (B) many one and into
Sol. (C) one one and onto (D) one one and into
Sol.

34. Let S be the set of all triangles and R+ be the set


of positive real numbers. Then the function, f : S→R+,
f(∆) = area of the ∆, where ∆ ∈ S is
(A) injective but not surjective
(B) surjective but not injective
(C) injective as well as surjective
(D) neither injective nor surjective
Sol.
 π
x : R →  0, 
–1 +
37. If f(x) = cot
 2
2
and g(x) = 2x – x : R → R. Then the range of the
function f(g(x)) wherever define is

 π  π π π π
(A)  0,  (B)  0,  (C)  ,  (D)  
 2  4 4 2  4
Sol.
35. Let ‘f’ be a function from R to R given by
x2 − 4
f(x) = . Then f(x) is
x2 + 1
(A) one-one and into (B) one-one and onto
(C) many-one and into (D) many-one and onto
Sol.

+ x
38. f(x) = |x – 1|, f : R →R ; g(x) = e , g : [–1, ∞)→R
If the function fog(x) is defined, then its domain and
range respectively are
(A) (0, ∞) & [0, ∞) (B) [–1, ∞) & [0, ∞)

 1  1 
(C) [–1, ∞) &  1 − ,∞ (D) [–1, ∞) &  − 1, ∞ 
 e   e 
Page # 36 FUNCTION

Sol. (A) fog is defined but gof is not


(B) gof is defined but fog is not
(C) both fog & gof are defined but they are unequal
(D) both gof & fog are defined and they are equal
Sol.

− 1 if x<0

x=0
39. Let g(x) = 1 + x – [x] and f(x) = 
0 if
,
 1 if x>0

then ∀ x, fog(x) equals
(where [ * ] represents greatest integer function).
(A) x (B) 1 (C) f(x) (D) g(x)
Sol.

42. If y = f (x) satisfies the condition

40. Let f: [0, 1] → [1, 2] defined as f(x) = 1 + x and  1 2 1


fx +  =x + 2 (x ≠ 0) then f(x) equals
g : [1, 2] → [0, 1] defined as g(x) = 2 – x then the  x x
composite function gof is 2
(A) – x + 2
2
(B) – x – 2
2
(C) x + 2
2
(D) x – 2
(A) injective as well as surjective Sol.
(B) Surjective but not injective
(C) Injective but non surjective
(D) Neither injective nor surjective
Sol.

43. If f(1) = 1 and f(n + 1) = 2f(n) + 1 if n ≥ 1, then


f(n) is equal to
n n n n–1
(A) 2 + 1 (B) 2 (C) 2 – 1 (D) 2 –1
41. Let f & g be two functions both being defined Sol.
x+ | x |
from R → R as follows f(x) = and
2

x for x<0
g(x) =  . Then
x
2
for x≥0
FUNCTION Page # 37

 1 + sin x 
47. The function f(x) = log   is
 1 − sin x 
(A) even (B) odd
44. A function f : R → R satisfies the condition,
2 4 (C) neither even nor odd (D) both even & odd
x f(x) + f(1 – x) = 2x – x . Then f(x) is
2 2 2 4 Sol.
(A) – x – 1 (B) –x + 1 (C) x – 1 (D) – x + 1
Sol.

45. A real valued function f(x) satisfies the functional


equation f(x – y) = f(x) f(y) – f(a – x) f(a + y) where
a is a given constant and f(0) = 1, f(2a – x) is equal to
(A) f(–x) (B) f(a) + f(a – x) (C) f(x) (D) –f(x)
Sol.
ax − 1
48. If the graph of the function f(x) = n x is
x (a + 1)
symmetric about y–axis, then n is equal to
(A) 2 (B) 2/3 (C) 1/4 (D) –1/3
Sol.

46. If f : R → R satisfies f(x + y) = f(x) + f(y), for all


n

x, y → R and f(1) = 7, then ∑ f (r ) is


r =1

7n 7(n + 1) 7n(n + 1)
(A) (B) (C) 7n(n+1) (D) .
2 2 2
Sol.

49. The graph of the function y = f(x) is symmetrical


about the line x = 2, then
(A) f(x + 2) = f(x – 2) (B) f(2 + x) = f(2 – x)
(C) f(x) = f(–x) (D) f(x) = –f(–x)
Page # 38 FUNCTION

Sol. Sol.

53. Fundamental period of f(x) = sec (sin x) is


(A) π/2 (B) 2π (C) π (D) a periodic
Sol.

50. If f(–x) = –f(x), then f(x) is


(A) neither odd nor even (B) an odd function
(C) an even function (D) periodic function
Sol.
54. If f(x) = sin [a] x has π as its fundamental period
then (where [ * ] denotes the gratest integer function)
(A) a = 1 (B) a = 9 (C) a ∈ [1, 2) (D) a ∈ [4, 5)
Sol.

 x 2 + 1
51. If g : [–2, 2] → R where g(x)=x +tan x + 
3 
 p 
be an odd function , then the value of the parameter
P is
55. The fundamental period of the function,
(A) –5 < P < 5 (B) P < 5 (C) P>5 (D) None of these
f(x) = x + a – [x + b] + sin πx + cos 2πx + sin 3πx +
Sol.
cos 4πx + ..... + sin (2n – 1) πx + cos 2 nπx
or every a, b ∈ R is
(where [ * ] denotes the greatest integer function)
(A) 2 (B) 4 (C) 1 (D) 0
Sol.

52. It is given that f(x) is an even function and satisfy the


π πx π
xf ( x 2 ) 56. The period of sin [x] + cos + cos [x],
relation f(x) = then the value of f(10) is 4 2 3
2 + tan 2 x.f ( x 2 )
where [x] denotes the integral part of x is
(A) 10 (B) 100 (C) 50 (D) None of these (A) 8 (B) 12 (C) 24 (D) Non–periodic
FUNCTION Page # 39

Sol. Sol.

59. Let f(x) = x (2 – x), 0 ≤ x ≤ 2. If the definition of ‘f’


is extended over the set, R – [0, 2] by f(x + 2) = f(x),
then ‘f’ is a
(A) periodic function of period 1
(B) non-periodic function
(C) periodic function of period 2
(D) periodic function of period 1/2
Sol.

57. The fundamental period of function


 1  2
f(x) = [x] +  x + + x +  – 3x + 15
 3   3
(A) 1/3 (B) 2/3 (C) 1 (D) Non–periodic 60. Let f(2, 4) → (1, 3) be a function defined by
Sol. x –1
f(x) = x –   , then f (x) is equal to
2
(where [ * ] denotes the greatest integer function)
x
(A) 2x (B) x +   (C) x + 1 (D) x – 1
2
Sol.

61. The mapping f : R → R given by


3 2
f(x) = x + ax + bx + c is a bijection if
2 2 2 2
(A) b ≤ 3a (B) a ≤ 3b (C) a ≥ 3b (D) b ≥ 3a
Sol.
58. Which one of the following is true.

16 x − 1
(A) f(x) = is an odd function
4x
(B) f(x) = sin |x| is an odd function
(C) if sin x + cos a x is periodic then ‘a’ is irrational
(D) if f1 (x), f2 (x) are periodic then their sum function
will always be periodic
Page # 40 FUNCTION

EXERCISE – II MULTIPLE CORRECT (OBJECTIVE QUESTIONS)

1. Let f : [–1, 1] → [0, 2] be a linear function which is 3. A function ‘f’ from the set of natural numbers to
onto then f(x) is/are
(A) 1 – x (B) 1 + x (C) x – 1 (D) x + 2 n − 1
 2 , when n is odd
Sol. integers defined by f(n)=  n is.
− , when n is even
 2

(A) one–one (B) many–one (C) onto (D) into


Sol.

2. In the following functions defined from [–1, 1] to


[–1, 1] the functions which are not bijective are

–1 2 –1
(A) sin (sin x) (B) sin (sin x)
π
x 3
(C) (sgn x ) ln e (D) x sgn x
Sol.  1− x 
4. Let f(x) =   , 0 ≤ x ≤ 1 and g(x) = 4x (1 – x),
 1+ x 
0 ≤ x ≤ 1. then
1 − 4x + 4x 2
(A) fog = ,0≤x≤1
1 + 4x − 4x 2
1 − 4x − 4x 2 1
(B) fog = , ≤x≤1
1 + 4x − 4x 2 2
8 x(1 − x )
(C) gof = ,0≤x≤1
(1 + x )2
8 x(1 + x )
(D) gof = ,0≤x≤1
(1 + x )2
Sol.
FUNCTION Page # 41

7. If f : R → [–1, 1], where f(x) = sin π/2 [x],


(where [*] dentoes the greatest integer function) then
(A) f(x) is onto (B) f(x) is into
(C) f(x) is periodic (D) f(x) is many one
Sol.

5. If ‘f’ and ‘g’ are bijective functions and gof is


defined then gof must be
(A) injective (B) surjective
(C) bijective (D) into only
Sol.

4 4 sin π[ x]
6. The period of the function f(x) = sin 3x + cos 3x is 8. If F(x) =
(A) π/6 (B) π/3 (C) π/2 (D) π/12 { x } , then F(x) is
Sol. (where {*} denotes fractional part of function and
[*] denotes greatest integer function)
(A) periodic with fundamental period 1
(B) even
(C) range is singleton
 { x} 

(D) identical to sgn  sgn
{ x} 
– 1,

(where { * } denotes fractional part of function
and [ * ] denotes greatest integer function and sgn
(x) is a signum function)
Sol.
Page # 42 FUNCTION

10. Which of the following functions are periodic ?


(A) f(x) = sgn (e–x)

1 if x is a rational number
(B) f(x) = 
0 if x is an irrational number

8 8
(C) f(x) = +
1 + cos x 1 − cos x
 1  1
(D) f(x) =  x +  +  x −  + 2 [–x]
 2   2 
(where [ * ] denotes greatest integer function)
Sol.

2
9. Function f(x) = sin x + tan x + sgn (x – 6x + 10) is
(A) periodic with period 2π
(B) periodic with period π
(C) Non–periodic
(D) periodic with period 4π
Sol.
FUNCTION Page # 43

EXERCISE – III SUBJECTIVE QUESTIONS


1. Find the domain of each of the following functions
 1 
(v) logx log2  
x − 5x + 3
3
 x − 1/ 2 
(i) f(x) =
x2 − 1 Sol.
Sol.

1
(ii) f(x) =
x+ | x |

Sol.

(vi) f(x) = 3 − 2 x − 21− x


Sol.
x + sin x
(iii) f(x) = e
Sol.

1
(iv) f(x) = + x+2
log10 (1 − x )
(vii) f(x) = 1− 1− x2
Sol.
Sol.
Page # 44 FUNCTION

2 –3/2
(viii) f(x) = (x + x + 1)
Sol.  5x − x 2 
(xii) f(x) = log1/ 4  

 4 
Sol.

x−2 1− x
(ix) f(x) = +
x+2 1+ x
Sol.

2
(xiii) f(x) = log10 (1 – log10(x – 5x + 16))
Sol.

(x) f(x) = tan x − tan 2 x


2. Find the range of each of the following functions
Sol.
(i) f(x) = |x – 3|
Sol.

x
(ii) f(x) =
1+ x2
Sol.

1
(xi) f(x) =
1 − cos x
Sol.
FUNCTION Page # 45

(iii) f(x) = 16 − x 2 sin x


+
cos x
Sol. (ix)
1 + tan x2
1 + cot 2 x
Sol.

| x − 4|
(iv) f(x) =
x−4
Sol.

(v) f(x) = 5 + 3 sin x + 4 cos x


Sol.

1
(vi) f(x) =
1+ x
Sol.

(x) f(x) = 1 – |x – 2|
Sol.

2
(vii) f(x) = 2 – 3x – 5x
Sol.

1
(xi) f(x) =
x−5
Sol.
(viii) 3 | sin x | – 4 | cos x |
Sol.
Page # 46 FUNCTION

1
(xii) f(x) =
2 − cos 3 x
Sol.

4 2
(xvi) f(x) = x – 2 x + 5
Sol.

x+2
(xiii) f(x) =
x − 8x − 4
2

Sol.
3
(xvii) f(x) = x – 12x, where x ∈ [–3, 1]
Sol.

2 4
(xviii) f(x) = sin x + cos x
Sol.
x 2 − 2x + 4
(xiv) f(x) =
x 2 + 2x + 4
Sol.

3. Find the domain and the range of each of the fol-


lowing functions
π2
(xv) f(x) = 3 sin − x2 1
16 (i) f(x) =
Sol. 4 + 3 sin x
Sol.
FUNCTION Page # 47

Sol.

(ii) f(x) = x!
Sol.

6. Solve the following equation for


x : 2x + 3[x] – 4 {–x} = 4
(where [ * ] & { * } denotes integral and fractional
part of x)
x2 − 9 Sol.
(iii) f(x) =
x −3
Sol.

2 3 2 3
(iv) f(x) = sin (x ) + cos (x )
Sol.

4x
4. If f(x) = , then show that f(x) + f(1 – x) = 1
4 +2
x

Sol.

7. Let f(x) be defined on [–2, 2] and is given by


 −1 , − 2 ≤ x ≤ 0
f(x) =  and g(x) = f(|x|) + |f(x)|,
x − 1 , 0 ≤ x ≤ 2
then find g(x).
Sol.

2
5. Draw the graph of the function f(x)=|x –4 | x |+3|
and also find the set of values of ‘a’ for which the
equation f(x) = a has exactly four distinct real roots.
Page # 48 FUNCTION

lnx
(iv) f(x) = x and g(x) = e
Sol.

9. Find whether the following functions are one–one


or many–one
2
(i) f(x) = |x + 5x + 6|
Sol.

(ii) f(x) = | log x |


Sol.

8. Check whether following pairs of functions are iden-


tical or not ?
2
(i) f(x) = x 2 & g(x) = ( x )
Sol.  π π
(iii) f(x) = sin 4x, x ∈  −, 
 8 8
Sol.

–1 –1
(ii) f(x) = sec (sec x) & g(x) = cosec (cosec x)
Sol.
1
(iv) f(x) = x + , x ∈ (0, ∞)
x
Sol.

1+ cos 2x
(iii) f(x) = & g(x) = cos x
2
Sol.
FUNCTION Page # 49

Sol.
1 
 −1
(v) f(x) =
1− e x 

Sol.

(ii) f(x) = x cos x


Sol.

1
(iii) f(x) =
sin | x |

Sol.

3x 2
(vi) f(x) = – cos πx (iv) tan (2 sin x)

Sol.
Sol.

11. Classify the following functions f(x) defined in


R → R as injective , surjective, both or none.
(i) f(x) = x | x |
Sol.
10. Let f : D → R where D is its domain. Find whether
the following functions are into/onto.

1+ x 6
(i) f(x) =
x3
Page # 50 FUNCTION

2
(ii) f(x) = x 13. Find fog and gof, if
x
Sol. (i) f(x) = e ; g(x) = log x
Sol.

x2
(iii) f(x) =
1+ x2 (ii) f(x) = |x| ; g(x) = sin x
Sol. Sol.

–1 2
(iii) f(x) = sin x ; g(x) = x
3 2
(iv) f(x) = x – 6 x + 11x – 6 Sol.
Sol.

2 1
(iv) f(x) = x + 2 ; g(x) = 1 – ,x≠1
1− x
Sol.

12. Let f : A → A where A = {x : – 1 ≤ x ≤ 1}. Find


whether the following function are bijective.
(i) x – sin x
Sol.
1 + x 2 x ≤1
14. If f(x)=  and g(x)=1–x ; –2 ≤ x ≤ 1
(ii) x |x|  x + 1 1< x ≤ 2
then define function fog(x).
Sol.
Sol.

πx
(iii) tan
4
Sol.

4
(iv) x
Sol.
FUNCTION Page # 51

1 + x, 0 ≤ x ≤ 2 18. If f(x + y) = f(x) . f(y) ∀ x, y ∈ N and f(1) = 2,


15. Let f(x) =  . Find fof..
3 − x, 2 < x ≤ 3
10

Sol. then find ∑ f (n) .


n =1
Sol.

2 +
16. If f(x) = ln (x – x+2) ; R →R and g(x) = {x}+1 ;
 x 2 + sin x 0 ≤ x < 1
[1, 2] → [1, 2], 19. If f(x) =  then extend the
−x
(where { * } denotes fractional part of x). Find the  x+e x ≥1
domain and range of f(g(x)) when defined. definition of f(x) for x ∈ (– ∞, 0) such that f(x) be-
Sol. comes
(i) An even function
Sol.

(ii) An odd function


Sol.

20. Examine whether the following functions are even


or odd or none.
(1 + 2 x )7
17. Let f(x) be a polynomial function satisfying the (i) f(x) =
2x
 1  1 Sol.
relation f(x) . f   = f(x) + f   x ∈ R – {0} and
x x ∀
f(3) = –26. Determine f′(1).
Sol.

sec x + x 2 − 9
(ii) f(x) =
x sin x
Sol.
Page # 52 FUNCTION

21. Find the period of the following functions


(iii) f(x) = 1 + x + x2 − 1 − x + x2 (where [ * ] denotes greatest integer function)
Sol.
(i) f(x) = 2 + 3 cos (x – 2)
Sol.

2
(ii) f(x) = sin 3x + cos x + |tan x|
Sol.
 x|x| , x ≤ −1

[1 + x] − [ x − 1] , − 1 < x < 1
(iv) f(x) = 
 −x|x| , x ≥1

Sol.

πx πx
2x(sin x + tan x ) (iii) f(x) = sin + sin
(v) f(x) = , 4 3
 x + 2π  Sol.
2  −3
 π 
where [ * ] denotes greatest integer function.
Sol.

3 2
(iv) f(x) = cos x – sin x.
5 7
Sol.

(v) f(x) = [sin 3x] + |cos 6x|


Sol.
FUNCTION Page # 53

1
(vi) f(x) = π
1 − cos x (iii) f(x) = tan [x],
2
Sol.
where [*] denotes greatest integer function
Sol.

sin12 x
(vii) f(x) =
1 + cos 2 6x
Sol. ln sin x 3
(iv) f(x) e + tan x – cosec (3 x – 5)
Sol.

1  | sin x | sin x 
 + 
(v) f(x) =
2  cos x | cos x | 
2 3
(viii) f(x) = sec x + cosec x Sol.
Sol.

22. Find the period of the following functions.


x x x
(vi) f(x) = sin x + tan + sin 2 + tan 3 + ........
sin2 x cos 2 x 2 2 2
(i) f(x) = 1 – −
1 + cot x 1 + tan x
Sol.
x x
+ sin + tan n
2n−1 2
Sol.

(ii) f(x) = log (2 + cos 3 x)


Sol.
Page # 54 FUNCTION

sin x + sin 3x
(vii) f(x) =
cos x + cos 3 x
Sol.

 π π
25. Let f : − ,  → B defined by
 3 6
23. Find the period of f(x) satisfying the condition 2 –1
2 3
(i) f(x + p) = 1 + {1 – 3f(x) + 3 f (x) – f (x)}
1/3 f(x) = 2 cos x + 3 sin2x + 1. Find B such that f exists.
–1
Sol. Also find f (x).
Sol.

(ii) f(x – 1) + f(x + 3) = f(x + 1) + f(x + 5)


Sol.

e 2 x − e −2 x
24. Let f : R → R be defined by f(x) = . Is
2
f(x) invertible ? If yes, then find its inverse.
Sol.

x–1
26. Let f : N → N where f(x) = x + (–1) find the
inverse of f.
Sol.
FUNCTION Page # 55

EXERCISE – IV ADVANCED SUBJECTIVE QUESTIONS

1. Find the domain of definitions of the functions


(Read the symbols [*] and {*} as greatest integers
and fractional part functions respectively.)

(i) f(x) = cos 2x + 16 − x 2


Sol.

1 − 5x
(iv) f(x) =
7−x − 7
Sol.

3 2
(ii) f(x) = log7 log5 log3 log2 (2x + 5x – 14x)
Sol. (v) y = log10 sin (x – 3) + 16 − x 2
Sol.

(iii) f(x) = ln ( x 2 − 5 x − 24 – x – 2)
 2 log10 x + 1 
(vi) f(x) = log100x  
Sol.  −x 
Sol.
Page # 56 FUNCTION

1 2
(vii) f(x) = + ln x(x – 1)
4x − 1
2

Sol.

(x) f(x) = ( x 2 − 3 x − 10).n2 ( x − 3)


Sol.

x
(viii) f(x) = log 1
2 x −1
2

Sol.

(xi) f(x) = log x (cos 2πx )


Sol.

1
(ix) f(x) = x2 − | x | +
9 − x2
Sol.
FUNCTION Page # 57

1 (xv) f(x) = logx sin x


cos x − Sol.
(xii) f(x) = 2
6 + 35 x − 6x 2
Sol.

  
  
  1 
(xvi) f(x) = log2  − log1/ 2  1 + +
 x°   
  sin  
   100   
log10 (log10 x ) − log10 ( 4 − log10 x ) − log10 3
Sol.

(xiii) f(x) = log1/ 3 (log4 ([ x ]2 − 5))


Sol.

1 1
2
(xiv) f(x) = + log(2{x}–5) (x – 3x + 10) +
[ x] 1− | x |

Sol.
Page # 58 FUNCTION

1
(xix) If f(x) = x 2 − 5 x + 4 & g(x) = x + 3, then
2 1 1
(xvii) f(x) = +log1 – {x}(x –3x+10)+ + f
[ x] 2− | x | sec(sin x )
find the domain of
g (x).
Sol.
Sol.

2. Find the domain & range of the following functions .

(xviii) f(x) = (5 x − 6 − x 2 )[{n{ x}}] +


(i) y = log 5 ( )
2 (sin x − cos x) + 3

−1
Sol.
 7 
(7x − 5 − 2x 2 ) +  n − x  
 2 
Sol.

2x
(ii) y =
1+ x2
Sol.

x 2 − 3x + 2
(iii) f(x) =
x2 + x − 6
Sol.
FUNCTION Page # 59

x
(iv) f(x) =
1+ | x |
Sol.

x+4 −3
(vii) f(x) =
x−5
Sol.

(v) y = 2 − x + 1+ x
Sol.

3. Find the set of real value(s) of p for which the


equation |2x + 5| + |2x – 5| = px + 10 has two
solutions.
Sol.

(vi) f(x) = log(cosec x – 1) (2 – [sin x] – [sin x]2)


Sol.
Page # 60 FUNCTION

4. A function f defined for all real numbers is defined


x , 0 ≤ x ≤1
as follows for x ≥ 0 : f(x) = [1, x > 1

How is f defined for x ≤ 0 if :


(i) f is even (ii) f is odd ?
Sol.

x
x −1
(ii) f(x) = 2
Sol.

5. The function f(x) is defined on the interval [0,1].


Find the domain of definition of the functions.
(a) f (sin x) (b) f (2x+3)
Sol.
10 x −10 − x
(iii) y =
10 x +10 − x
Sol.

1  3 
6. A function f :  ,∞  →  ,∞  defined as,
2  4 
f(x) = x2 − x+1 . Then solve the equation f(x) = f −1 (x). 8. Write explicitly, functions of y defined by the
Sol. following equations and also find the domains of
definition of the given implicit functions
(i) 10x + 10y = 10
Sol.

7. Compute the inverse of the functions (ii) x + y= 2y


Sol.
(i) f(x) = ln  x + x +1 
2
 
Sol.
FUNCTION Page # 61

9. Find whether the following functions are even or


x x
odd or none (g) f(x)= + +1
e −1 2
x
 2 
(a) f(x) = log x + 1 + x 
  Sol.
Sol.

x(a x + 1)
(b) f(x) =
ax − 1
Sol.

(h) f(x) = [(x+1)²]1/3 + [(x −1)²]1/3


Sol.

(c) f(x) = sin x + cos x 10. Solve the following problems from (i) to (v) on
Sol. functional equation.
(i) The function f(x) defined on the real numbers has
the property that f(f(x)) . (1 + f(x)) = –f(x) for all x in
the domain of f. If the number 3 is the domain and
range of f, compute the value of f(3).
(d) f(x) = x sin2 x − x3 Sol.
Sol.

(e) f(x)= sin x − cos x


Sol.

(ii) Suppose f is a real function satisfying


f(x + f(x)) = 4f(x) and f(1) = 4. Find the value of f(21).
(1 + 2 x )2
(f) f(x) = Sol.
2x
Sol.
Page # 62 FUNCTION

(iii) Let ‘f’ be a function defined from


R+ → R+. [f(xy)]2 = x(f(y))2 for all positive numbers
x and y and f(2) = 6, find the value of f(50).
Sol.

(iv) Let f(x) be a function with two properties 12. A function f, defined for all x, y ∈ R is such that
(a) for any two real number x and y, f(x + y)=x + f(y) f(1) = 2 ; f(2) = 8 & f(x + y) – k x y = f(x) + 2 y2,
and (b) f(0) = 2. Find the value of f(100). where k is some constant. Find f(x) & show that :
Sol.  1 
f(x + y) f   = k for x + y ≠ 0.
x+ y
Sol.

(v) Let f(x) be function such that f(3) = 1 and


f(3x) = x + f(3x – 3) for all x. Then find the value of
f(300).
Sol.
13. Prove that the function defined as, f(x) =
 1
e − |ln{ x }|
− { x}
|ln{ x }|
where ever it exists

 {x} otherwise, then
f(x) is odd as well as even.
(where {x} denotes the fractional part function)
Sol.

11. Let ‘f’ be a real valued function defined for all real
numbers x such that for some positive constant ‘a’

the equation f (x + a ) = + f (x) − (f (x)) holds for all x .


1 2
2
Prove that the function f is periodic.
Sol.
FUNCTION Page # 63

9x
14. Let f(x) = then find the value of the sum f
9x + 3
 1   2   3   2005 
  +f   +f   + .... + f  .
 2006   2006   2006   2006 
Sol.

 1– x 
16. A function f : R → R is such that f   = x for
 1+ x 
all x ≠ – 1. Prove the following.
Sol.

(a) f (f (x)) = x
Sol.

(b) f (1/x) = – f (x), x ≠ 0


Sol.

15. The set of real values of ‘x’ satisfying the equality


3 4
 x  +  x  = 5 (where [ * ] denotes the greatest (c) f (–x – 2) = – f (x) – 2.
   
Sol.
 b
integer function) belongs to the interval  a,  where
 c
b
a, b, c ∈ N and is in its lowest form. Find the value
c
of a + b + c + abc.
17. If f(x) = max (x, 1/x) for x > 0 where max (a, b)
Sol.
denotes the greater of the two real numbers a and b.
Define the function g(x) = f(x).f (1/x) and plot is graph.
Sol.
Page # 64 FUNCTION

(b) If a,b are positive real numbers such that a–b= 2,


then find the smallest value of the constant L for

which x 2 + ax − x 2 + bx < L for all x > 0.


Sol.

18. Let f be a one-one function with domain {x, y, z}


and range {1, 2, 3}. It is given that exactly one of
the following statements is true and the remaining (c) Let f(x) = x2 + kx ; k is real number. The set of
two are false. f(x) = 1 ; f(y) ≠ 1 ; f(z) ≠ 2. Determine values of k for which the equation f(x) = 0 and f(f(x)) = 0
f–1(1) have same real solution set.
Sol. Sol.

(d) Let P(x) = x6 + ax5 + bx4 + cx3 + dx2 + ex + f be a


polynomial such that P(1) = 1 ; P(2) = 2; P(3) = 3 ;
P(4) = 4; P(5) = 5 and P(6) = 6 then find the value of
19. (a) A function f is defined for all positive integers P(7).
and satisfies f(1) = 2005 and Sol.
f(1) + f(2) + ...+ f(n) = n2f(n) for all n > 1. Find the
value of f(2004)
Sol.

(e) Let a and b be real numbers and let


f(x) = a sin x + b3 x + 4, ∀ x ∈ R . If f(log10(log310)) = 5

then find the value of f (log10 (log10 3))


Sol.
FUNCTION Page # 65

Sol.
20. Let [x] = the greatest integer less than or equal
to x. If all the values of x such that the product
 1  1
x – 2  x + 2 
is prime, belongs to the set
 
[x1, x2) ∪ [x3, x4), find the value of x12 + x 22 + x 23 + x 24 .
Sol.

23. Let f(x) = x135 + x125 – x115 + x5 + 1. If f(x) is


divided by x3 – x then the remainder is some function
of x say g (x). Find the value of g(10).
Sol.

24. A is a point on the circumference of a circle.


Chords AB and AC divide the area of the circle into
21. Suppose p(x) is a polynomial with integer
three equal parts. If the angle BAC is the root of the
coefficients. The remainder when p(x) is divided by x – 1
equation, f(x) = 0 then find f(x).
is 1 and the remainder when p(x) is divided by x – 4 is
Sol.
10. If r(x) is the remainder when p(x) is divided by
(x – 1) (x – 4), find the value of r (2006).
Sol.

22. Let f : R → R – {3} be a function with the property


that there exist T > 0 such that
f ( x) – 5
f (x + T) = for every x ∈ R. Prove that f(x) is
f ( x) – 3
periodic.
Page # 66 FUNCTION

EXERCISE – V JEE PROBLEMS

1. If the function f : [1, ∞) → [1, ∞) is defined by


– 1 , x < 0
f(x) = 2x(x – 1), then f–1(x) is [JEE 99, 2] 
4. (a) Let g(x) = 1 + x – [x] & f(x) =  0 , x = 0 .
x ( x −1) 1 , x>0
 1 1 
(A)   (B) (1 + 1 + 4 log2 x ) Then for all x, f(g(x)) is equal to [JEE 2001 (Scr.), each 1 mark]
2 2
(A) x (B) 1 (C) f(x) (D) g(x)
where [ * ] denotes the greatest integer function.
1
(C) (1 – 1 + 4 log2 x ) (D) not defined
2
Sol.
Sol.

1
(b) If f : [1, ∞) → [2, ∞) is given by, f(x) = x + ,
x
then f–1(x) equals.

x + x2 − 4 x x − x2 − 4
(A) (B) 2 (C) (D) 1– x 2 − 4
2. The domain of definition of the function, y(x) given 2 1+ x 2
by the equation, 2x + 2y = 2 is [JEE 2000(Scr.), 1] Sol.
(A) 0 < x ≤ 1 (B) 0 ≤ x ≤ 1 (C) – ∞ < x ≤ 0 (D) – ∞ < x < 1
Sol.

3. Given X = {1, 2, 3, 4}, find all one–one, onto


mappings, f : X → X such that, f(1) = 1, f(2) ≠ 2 and
f(4) ≠ 4 [REE 2000, 3]
Sol.

log2 ( x + 3)
(c) The domain of definition of f(x) = is
x 2 + 3x + 2
(A) R \ {–1, – 2} (B) (–2, ∞)
(C) R \ {–1, –2, –3} (D) (–3, ∞) \ {–1, –2}
FUNCTION Page # 67

Sol. Sol.

(d) Let E = {1, 2, 3, 4} & F = {1, 2}. Then the


number of onto functions from E to F is
(A) 14 (B) 16 (C) 12 (D) 8
Sol. (b) Let function f : R → R be defined by
f(x) = 2x + sinx for x ∈ R. Then f is
(A) one to one and onto (B) one to one but NOT onto
(C) onto but NOT one to one (D) neither one to one nor onto
Sol.

αx
(e) Let f(x) = , x ≠ – 1. Then for what value of α
x +1
is f(f(x)) = x ?
(A) 2 (B) – 2 (C) 1 (D) – 1 x
6. (a) Let f(x) = defined from (0, ∞) → [0, ∞)
Sol. 1+ x
then by f(x) is [JEE. 2003 (Scr.), 2+2]
(A) one - one but not onto (B) one- one and onto
(C) Many one but not onto (D) Many one and onto
Sol.

5. (a) Suppose f(x) = (x + 1)2 for x ≥ – 1. If g(x) is the


function whose graph is the reflection of the graph of
f(x) with respect to the line y = x, then g(x) equals
[JEE. 2002 (Scr.), 3 + 3]
1 x2 + x + 2
, x ≥ –1 (b) Range of the function f(x) = is
(A) – x – 1, x ≥ 0 (B) x2 + x + 1
( x + 1) 2
 7  7
(C) x + 1, x ≥ −1 (D) x – 1, x ≥ 0 (A) [1, 2] (B) [1, ∞) (C) 2,  (D) 1,. 
 3  3
Page # 68 FUNCTION

Sol.

10. Let f(x) = x2 and g(x) = sin x for all x ∈ R. Then the
set of all x satisfying (f o g o g o f) (x) = g(g o g of) (x),
where (f o g) (x) = f(g(x)), is
(A) ± nπ , n ∈ {0, 1, 2, ...} (B) ± nπ , n ∈ {1, 2, ...}
7. Let f(x) = sinx + cosx, g (x) = x2 – 1. Thus g(f(x)) is
invertible for x ∈ [JEE 2004 (Scr.), 1] π
(C) + 2nπ, n ∈ {......, –2, –1, 0, 1, 2 .....}
2
 π   π   π π  π
(A)  – , 0 (B)  – , π (C)  – ,  (D) 0,  (D) 2nπ, n ∈ {......, –2, –1, 0, 1, 2, ....} [JEE 2011]
 2   2   4 4  2
Sol.
Sol.

11. The function f : [0, 3] → [1, 29], defined by


f(x) = 2x3 – 15x2 + 36x + 1, is [JEE 2012]
8. If the functions f(x) & g(x) are defined on R → R such that (A) one-one and onto. (B) onto but not one-one.
(C) one-one but not onto. (D) neither one-one nor onto
 0, x ∈ rational 0, x ∈ irrational
f ( x) =  , g( x ) =  Sol.
x, x ∈ irrational  x, x ∈ rational

then (f – g) (x) is [JEE 2005 (Scr.), 1]


(A) one – one and onto (B) neither one–one nor onto
(C) one-one but not onto (D) onto but not one-one
Sol.
2
12. Let f : (–1, 1) → IR be such that f(cos 4θ) =
2 − sec2 θ
 π π π 1
for θ ∈  0,  ∪  ,  . Then the value(s) for f   is (are)
 4 4 2 3

3 3 2 2
(A) 1 – (B) 1 + (C) 1 – (D) 1 +
2 2 3 3
9. Let S = {1, 2, 3, 4}. The total number of unordered Sol. [JEE 2012]
pairs of disjoint subsets of S is equal to [JEE 2010]
(A) 25 (B) 34 (C) 42 (D) 41
Sol.
FUNCTION Page # 69

Answer Ex–I SINGLE CORRECT (OBJECTIVE QUESTIONS)

1. D 2. D 3. B 4. A 5. A 6. A 7. D 8. B

9. B 10. D 11. D 12. A 13. C 14. B 15. D 16. C

17. C 18. B 19. A 20. A 21. B 22. C 23. B 24. C

25. B 26. D 27. A 28. A 29. C 30. C 31. D 32. D

33. D 34. B 35. C 36. D 37. C 38. B 39. B 40. A

41. D 42. D 43. C 44. B 45. D 46. D 47. B 48. D

49. B 50. B 51. C 52. D 53. C 54. D 55. A 56. C

57. A 58. A 59. C 60. C 61. B

Answer Ex–II MULTIPLE CORRECT (OBJECTIVE QUESTIONS)

1. A,B 2. B,C,D 3. A,C 4. A,C 5. A,B,C 6. A,B,C 7. B,C,D

8. B,C,D 9. A,D 10. A,B,C,D

Answer Ex–III SUBJECTIVE QUESTIONS

1   3
1. (i) R – {–1, 1} (ii) (0, ∞) (iii) R (iv) [–2, 0) ∪ (0, 1) (v)  ,1 ∪  1, 
2   2
 π
(vi) [0, 1] (vii) [–1, 1] (viii) R (ix) φ (x) ∪ nπ, nπ + 4 
n∈I
(xi) R – {2nπ}, n ∈ I (xii) (0, 1) ∪ [4, 5) (xiii) (2, 3)

 1 1
2. (i) [0, ∞) (ii) − ,  (iii) [0, 4] (iv) {–1, 1} (v) [0, 10]
 2 2

49
(vi) (0, 1] (vii) (–∞, ] (viii) [–4, 3] (ix) [–1, 1] (x) (– ∞, 1]
20

1   1  1  1 
(xii)  ,1 (xiii)  − ∞,− ∪ − ,∞  (xiv)  ,3
+
4   20 
(xi) R
3   3 

 3  3 
(xv) 0,  (xvi) [4, ∞) (xvii) [–11, 16] (xviii)  ,1
 2 4 
Page # 70 FUNCTION

1
3. (i) Domain : R, Range : ≤y≤1 (ii) Domain : N ∪ {0}, Range : (n! : n = 0, 1, 2,.....}
7
(iii) Domain : R – {3}, Range : R – {6} (iv) Domain : R, Range : {1}
(0, 3)

y=1
a ∈ (1, 3) ∪ {0} 3   − x , −2 ≤ x < 0
5. 6.   7. g(x) =  0 , 0 ≤ x ≤1
2 2(x − 1) , 1 < x ≤ 2

8. (i) No. (ii) Yes (iii) No (iv) No

9. (i) many–one (ii) many–one (iii) one–one (iv) many–one (v) one–one (vi) many–one

10. (i) into (ii) onto (iii) into (iv) onto

11. (i) bijective (injective as well as surjective) (ii) neither injective nor surjective

(iii) neither surjective nor injective (iv) surjective but not injective

12. (i) No (ii) Yes (iii) yes (iv) No

13. (i) fog = x, x > 0 ; gof = x, x ∈ R (ii) |sin x|, sin |x|

–1 2 –1 2
3x 2 − 4x + 2 x 2 + 2
(iii) sin (x ), (sin x ) (iv) , 2
(1 − x )2 x +1

2 − 2 x + x 2 2 + x , 0 ≤ x ≤ 1
0 ≤ x ≤1 
14. f(g(x)) =  15. (fof) (x) = 2 − x , 1 < x ≤ 2
 2−x − 1≤ x < 0 4 − x , 2 < x ≤ 3

16. Domain : [1, 2] ; Range : [ln2, ln4) 17. –3 18. 2046

 x 2 − sin x − 1 < x ≤ 0  − x 2 + sin x −1 < x ≤ 0


19. (i) f(x) =  (ii) f(x) = 
 −x+e x ≤ −1
x x
 x−e x ≤ −1

20. (i) neither even nor odd (ii) even (iii) odd (iv) even (v) odd


21. (i) 2π (ii) 2π (iii) 24 (iv) 70π (v) (vi) 2π
3
(vii) π/2 (viii) 2π

2π n
22. (i) π (ii) (iii) 2 (iv) 2π (v) 2π (vi) 2 π (vii) π
3

–1 –1 1
23. (i) 2p (ii) 8 24. f : R → R, f (x) = ln (x + x2 + 1 )
2

1  −1 x − 2  π 
25. B = [0, 4]; f
–1
(x) =  sin   −  26. f
–1
(x) = x + (–1)
x–1
,x∈N
2   2  6
FUNCTION Page # 71

Answer Ex–IV ADVANCED SUBJECTIVE QUESTIONS

 5π −3π   π π  3π 5π   1
1. (i)  − 4 , 4  ∪  − 4, 4  ∪  4 , 4  (ii)  −4,− 2  ∪ (2, ∞) (iii) (– ∞, – 3]
       

(iv) (–∞, – 1) ∪ [0, ∞) (v) (3 − 2π < x < 3 − π) U (3 < x ≤ 4)

 1   1 1 
(vi)  0,  ∪  ,  (vii) (−1 < x < −1/2) U (x > 1)
 100   100 10 

1 − 5   1 + 5 
(viii)  ,0  ∪  ,∞  (ix) (−3, −1] U {0} U [1,3)
  2 
 2   

(x) {4} ∪ [5, ∞) (xi) (0 , 1/4) U (3/4 , 1) U {x : x ∈ N, x ≥ 2}

 1 π   5π 
(xii)  −,  ∪  ,6  (xiii) [–3, –2) ∪ [3, 4) (xiv) φ
 6 3  3 

(xv) 2Kπ < x < (2K + 1)π but x ≠ 1 where K is non−negative integer

(xvi) {x 1000 ≤ x < 10000} (xvii) (–2, –1) ∪ (–1, 0) ∪ (1, 2)

 5
(xviii) (1, 2) ∪  2,  (xix) (− ∞ , −3) ∪ (−3 , 1] ∪ [4 , ∞)
 2

2. (i) D : x ∈ R R : [0 , 2] (ii) D = R ; range [–1,1]

(iii) D : {xx ∈ R ; x ≠ −3 ; x ≠ 2} ; R : {f(x)f(x) ∈R , f(x) ≠ 1/5 ; f(x) ≠ 1}

(iv) D : x ∈ R – {–1, 1} ; range ∈ R – {–1, 1} (v) D : x ∈ [–1, 2] ; range ∈ (–∞, 3]

π π 5π
(vi) D : x ∈ (2nπ, (2n + 1)π) – {2nπ + , 2nπ + , 2nπ + , n ∈ Ι} and
6 2 6
R : loga 2 ; a ∈ (0, ∞) –{1} ⇒ Range is (–∞, ∞) – {0}

 1  1 1
(vii) D : [–4, ∞) – {5} ; R :  0,  ∪  , 
6  
6 3 
3. p ∈ (–4, 4) – {0}

4. (i) f(x) = 1 for x < – 1 & –x for –1 ≤ x ≤ 0; (ii) f(x) = –1 for x < – 1 and x for –1 ≤ x ≤ 0.
Page # 72 FUNCTION

5. (a) 2Kπ ≤ x ≤ 2Kπ + π/2 where K ∈ I (b) [−3/2 , −1]

e x −e − x log2 x 1 1+ x
6. x =1 7. (i) (ii) (iii) log
2 log2 x−1 2 1− x
8. (i) y = log (10 − 10x) , − ∞ < x < 1 (ii) y = x/3 when − ∞ < x < 0 & y = x when 0 ≤ x < + ∞

9. (a) odd (b) even (c) neither odd nor even (d) odd (e) neither odd nor even

(f) even (g) even (h) even

10. (i) –3/4, (ii) 64 (iii) 30 (iv) 102 (v) 5050

1
if 0 < x ≤ 1
12. f(x) = 2x 2
14. 1002.5 15. 20 17. g( x ) =  x 2
 2
 x if x > 1

1
18. f–1(1) = y 19. (a) , (b) 1, (c) [0, 4), (d) 727, (e) 3
1002

π
20. 11 21. 6016 23. 21 24. f(x) = sinx + x –
3

Answer Ex–V JEE PROBLEMS

1. B 2. D

3. {(1,1), (2, 3), (3, 4), (4, 2)} ; {(1,1), (2, 4), (3, 2), (4, 3)} and {(1,1), (2, 4), (3, 3), (4, 2)}

4. (a) B, (b) A, (c) D, (d) A, (e) D 5. (a) D, (b) A 6. (a) A, (b) D

7. C 8. A 9. D 10. A 11. B 12. A,B


MATH BOOK

FUNCTION

Solution Slot

Short Book
JEE Main | CBSE

BY APARNA DWIVEDI
Solution Slot – 1 (Mathematics) Page # 1

FUNCTION

EXERCISE – I HINTS & SOLUTIONS


Sol.1 D Sol.4 A
Domains of f(x) is (– ∞, 0]
–log0.3 (x – 1) Domains of f (6{x} 2 – 5 {x} + 1)
f(x) =
x2 + 2x + 8 1 1
6{x}2 – 5{x} + 1 ≤ 0 ⇒ ≤ {x} ≤
x–1>0 ⇒ x> 1 3 2
& x + 2x – 8 > 0 ⇒ D < 0
2
⇒ x∈R
& –log0.3 (x – 1) ≥ 0 ⇒ log0.3 (x – 1) ≤ 0 1 1  1 1
⇒n+ ≤ x ≤ n + ; n ∈ I ⇒ nU∈I n + ,n + 
⇒ x – 1 ≥ 1 ⇒ x ≥ 2 ∴ Domain x ∈ [2, ∞) 3 2  3 2 

Sol.2 D Sol.5 B

  1    2x – 1 
f(x) = log1/2  – log2 1 +  – 1 f(x) = – log x + 4  log2
3 + x 
  4 x   2

 1  x+4
⇒ – log2  1 + 1/ 4  – 1 > 0 Case-1 : > 1 ⇒ x > –2
 x  2

 1  log x + 4  2x – 1 
⇒ – log2  1 + 1/ 4  > 1 ; – log2   ≥0
 x 
x> 0 2  x+3 

 1  1 1 log x + 4  2x – 1 
⇒ log2  1 + 1/ 4  < –1 ⇒1 + 1/ 4 < ⇒ log2   ≤0
 x  x 2 2  x+3 

1 1  2x – 1  2x – 1
⇒ 1/ 4 < – ⇒ x∈ φ ⇒ log2   ≤1 ⇒ ≤2
x 2  x+3  x+3

7
Sol.3 B ⇒ ≥0 ⇒ x > –3
x+3
q2 – 4pr = 0, p > 0
f(x) = log (px3 + (p + q) x2 + (q + r) x + r) 2x – 1 1
⇒ px3 + (p + q) x2 + (q + r) x + r > 0 & >0 ⇒ x> ; x < –3
x+3 2
⇒ (px3 + px2) + (qx2 + qx) + (rx + r) >0
⇒ px2 (x + 1) + qx (x + 1) + r( x + 1) > 0  2x – 1  2x – 1
& log2   >0 ⇒ >1
⇒ (x + 1) (px2 + qx + r) > 0 ⇒ D =q2 – 4pr  x+3  x+3
Means it is perfect square
x–4
b ⇒ >0 ⇒ x > 4 ; x < –3
x=– x+3
2a
2
 q  –q
(x + 1)  x +  >0 ⇒ x + 1 > 0 ⇒ x > –1 (x ≠ 2p )
 2p 

 q –3 –2 ½ 4
⇒ x ∈ (–1, ∞) – – 
 2p  ∴ x ∈ (4, ∞) .......(i)

  q  x+4
∴ x ∈ R – (– ∞,–1) ∪  – 2p   Case-2 : 0 < <1 ⇒ –4<x<–2
  2

2x − 1
– log x + 4 log2 ≥0
x+3
2
Page # 2 Solution Slot – 1 (Mathematics)

2x – 1 2x – 1 ⇒1≤ 2 (sin x– cos x) + 3 ≤ 5


⇒ log x + 4 log2 ≤ 0 ⇒ log2 ≥1
x+3 x+3
2 ⇒ 0 ≤ log 5 [m] ≤ 2 ∴ Range ∈ [0, 2]
7
⇒ ≤0 ⇒ x < –3
x+3 Sol.10 D

2x – 1 1 f(x) = log 2 (2 – log2 (16 sin2 x + 1))


& >0 ⇒ x> ; x < –3
x+3 2 ⇒ 0 ≤ sin2 x ≤ 1 ⇒ 0 ≤ 16 sin2 x ≤ 16
⇒ 1 ≤ (16 sin – x + 1) ≤ 17
2

 2x – 1  x–4 ⇒ 0 ≤ log2 (16 sin2 x + 1) ≤ log217


& log2   >0 ⇒ > 0
 x+3  x+3 ⇒ – log217 ≤ – log2 (16 sin2 x + 1) ≤ 0
⇒ x > 4 ; x < –3 ∴ x ∈ (–4, – 3) ...(2) ⇒ 2 – log217 ≤ 2 – log2 (16 sin2 x+ 1) ≤2
(1) ∪ (2) ⇒ x ∈ (–4, –3) ∪ (4, ∞) ⇒ log 2 (2 – log 17) ≤ log 2 M ≤ 2
2

Sol.6 A ∴ – ∞ < log 2


M≤2

log1/ 3 log4 ([ x ]2 – 5 )
Sol.11 D
⇒ log1/3 log4 ([x]2 – 5) ≥ 0 ⇒ 0 < log4 ([x]2 – 5) ≤ 1 [2 cos x] + [sin x] = – 3
⇒ 1 < [x]2 – 5 ≤ 4 ⇒ 6 < [x]2 ≤ 9
[x] always gives integer value so square of GTF
will also give Integer value. In between 6 and 9
are only perfect square value possible. –2, –1, 0, 1,2 –1, 0, 1
[x]2 = 9 ⇒ [x] = 3 [x] = – 3
3 ≤ x< 4 – 3 ≤ x< – 2
∴ x ∈ [–3, –2) ∪ [3, 4)
–3
Sol.7 D – 2 ≤ 2 cos x< – 1
2π / 3 O
f(x) = (x12 – x9 + x4 – x + 1)–1/2 1
1 –
↓ ↓ ↓ ↓ ↓ – 1 ≤ cos x < – 2
2
If x < 0 +ve –(–ve) +ve –(–ve) +ve > 0
If x= 0 f(x) > 0  2π 4 π  –1 +1
If x > 0 x9 (x3 – 1) + x(x3 – 1) + 1 x∈  , 
 3 3  1
↓ ↓ –
Now if x ≥ 1 +ve +ve + 1 >0 – 1 ≤ sinx < 0 2
4π / 3 O
If 0 < x < 1 x ∈ (π, 2π)
1 – x + x4 – x9 + x12
(1 – x) + x4 (1 – x5) + x12 > 0 ⇒ x ∈ R
0 2π π 4π 2π
Sol.8 B 3 3
f(x) = 4x + 2x + 1 (0, 1)

Let 2x = t ; t ∈ (0, ∞) 4π
∴ π<x<
= t2 + t + 1 3

2 2 f(x) = sin x + 3 cos x


 1 1  1 3
f(x) =  t +  – + 1 =  t +  + = (1, ∞)
 2 4  2 4 1 3   π
= 2  2 sin x + 2 cos x  = 2 sin  x +  = 2 sin θ
   3
Sol.9 B
π π 4π
f(x) = log 5 { 2 (sin x – cos x) + 3} π+ <θ< +
3 3 3
⇒ – 2 ≤ sin x – cos ≤ 2 4π 5π
4π 5π 3 3
⇒ <θ<
⇒ –2≤ 2 (sin x – cos x) ≤ 2 3 3 −
3
2
Solution Slot – 1 (Mathematics) Page # 3

Sol.15 D
3
⇒ – 1 ≤ sin θ < –
2  e x − e− x
e x – e –|x|
 ; x≥0
f(x) = x =  2e x
⇒ – 2 ≤ 2 sin θ < – 3 ∴ [– 2, – 3) e + e|x| 
 0 ; x<0

Sol.12 A for x ≥ 0
f(x) = 7 – x px – 3 n
Pr n ∈ N
7 – x > 0 ⇒ x <7 r∈w
ex – e– x e 2x – 1
y= =
x–3≥ 0⇒x≥ 3 n≥r 2e x 2e2x
7–x≥ x– 3⇒x≤ 5
x ∈ {3, 4,5} x = 3 4P0 = 1 t2 – 1
Let t = ex ⇒ y =
x = 4 3P1 = 3 2t 2
x = 5 2P2 = 2 ex ≥ 1 ⇒ t≥1
∴ 2yt = t – 1
2 2

Sol.13 C
1 1
t2 ≥ 1 ⇒ t2 = ≥ 1 ⇒ 0≤y<
cos
x
1 1
1 – 2y 2
2
x x x
f(x) = 1 cos – cos = 2 + 2 cos2
2 2 2 Sol.16 C
x
– cos 1 –1
2
sin2 x + 4 sin x + 5
f(x) =
2 sin2 x + 8 sin x + 8
x x
0 ≤ cos2 ≤1 ⇒ 0 ≤ 2 cos2 ≤2
2 2  2 sin2 x + 8 sin x + 10 
1
f(x) =  2

⇒ 2 ≤ 2 + 2 cos2
x
≤4 ∴ 2≤y≤4
2  2 sin x + 8 sin x + 8 
2
1  2 
= 1 + 2 
Sol.14 B 2 2  2 sin x + 8 sin x + 8 
Area (∆AMN) D C

1 1  1 
= (2x) x = x2 = 1 + 2
2
P
2  (sin x + 2) 
2
N
(AP = 2) x 1
f(x)|max. = [1 + 1] = 1
0<x≤ 2 x x 2
A B
0<x ≤22
M
1  1 5 5 
f(x)|min. = 1+ = ∴ Range ∈  , 1
2 ≤x<2 2 D N C 2  9  9 9 

PC = 2 2 – x
Sol.17 C
M
MN = 2(2 2 – x) [x] + 2 {– x} = 3x
Area (∆AMN) Case–1 : x ∈ I ; x + 0 = 3x ⇒ 2x = 0 ⇒ x = 0
Case–2 : x ∉ I
1
= 2(2 2 – x) x ⇒ [x] +2 (1 – {x}) = 3x ⇒ [x] + 2 – 2 (x – [x]) = 3x
2 A B ⇒ [x] + 2 – 2x + 2 [x] = 3x ⇒ 3[x] = 5x – 2 ...(1)
2 2
= 2 2 x – x = – (x – 2 2 x) & x – {x} + 2 – 2{x} = 3x
2 2
⇒ 2 – 3{x} = 3x ⇒ 0 ≤ {x} < 1
= – [(x – 2 ) – 2] = 2 – (x – 2) ⇒ 0 ≤ 3{x} < 3 ⇒ 0 ≤ 2 – 2x < 3
⇒ x = 2 , y = 2 ; x = 2 2 , y = 0 ∴ y ∈ (0, 2] ⇒ – 2 ≤ – 2x < 1 ⇒ – 1 < 2x ≤ 2
1
∴ – < x ≤1
2
Page # 4 Solution Slot – 1 (Mathematics)

of p and minimum value of q = 2 – 0 = 2


1
If – <x<0
2
Sol.23 B
Domain Co-domain
1 z → integer
from (1) ⇒ – 3 = 5x – 2 ⇒ x=– f(x) = ax2 + bx+ c f(1) z z
5
=a+b+c=I
If 0 ≤ x< 1
f(1) – f(2) = a + b ∈ I
from (1) ⇒ 0 = 5x – 2 ⇒ x = 2/5 ∴ 3 solution
f(0) = c ∈ I
If x = 1 (reject as x ∈ I is already taken)

Sol.24 C
Sol.18 B
[sin–1 x] = x – [x] (A) e( nx ) / 2 and x ⇒ D1 ∈ (0, ∞) ; D2 ∈ [0, ∞)
[sin–1 x] = {x} Domain are not same so not identical
–2, –1, 0, 1 0 ≤ {x} < 1 (B) tan–1 (tan x) and cot–1 (cot x)
possible only if [sin–1 x] = 0 & {x} = 0 Domain are not same so non identical
0 ≤ sin–1 x < 1 ; x=0 (C) cos2x + sin–1x and sin2x + cos4x
0 ≤ x < sin 1 ⇒ x ∈ R & x ∈ R Identical
common solution is x = 0
Sol.25 B
Sol.19 A f : [2, ∞) → y
f(x) = x2 – 4x + 5
 1 1999  1
 2 + 2000  = [0.5+0.995] = 1 one – one and onto
 
–D (16 − 20)
 =– =1
4a 4
 1 1000  for one-one and onto y = 1 ∴ range [1, ∞)
 2 + 2000  = [0.5 + 0.5] = 1
 
Sol.26 D
(1 + 1 + .......... + 1)1000 times = 1000
2x 2 – x + 5 g( x )
Sol.20 A f(x) = 2 =
7 x + 2x + 10 h( x )
f(x) = sgn [x + 1]
=1 if [x + 1] > 0 –1 Quadratic expression g(x) & h(x) are always posi-
⇒ [x] > – 1 ∴ x ≥ 0 –1 tive so f(x) is always positive.
=0 if [x + 1] = 0 so range ≠ co-domain.
⇒ [x] = – 1 ∴ –1 ≤ x < 0 –1
= – 1 if [x + 1] < 0 Sol.27 A
⇒ [x] < – 1 ∴ x < –1 f : R → R f(x) = x3 +x2 + 3x + sin x
2
Sol.21 B
f ’(x) = 3x
 +
2x +
3 + cos x > 0∀x ∈ R
1 ( −1,1)
f(x) = 2 sin2θ + 4 cos (x + θ) sinx.sinθ + cos (2x+2θ)
f(x) = cos 2x –D ( 4 – 36) 32
=– = = 2.57
4a 12 12
π   π 
⇒ f  – x  = cos  2  4 – x   = sin 2x f(x) = one-one, Range = Co-domain
4    
Sol.28 A
π 
⇒ f2(x)+ f2  – x  = 1 4a – 7 3 2
4  f(x) = x + (a – 3 )x + x + 5
3

Sol.22 C 7
Case – 1 : a =
0 < A < 1, 0 < B < 1, 0 < c < 1 4
⇒ 0<A+B +C <3
p = [A + B + C] = 2, q = [A] + [B] + [C] = 0 5 2
f(x) = – x + x+ 5 which can't be one-one
Maximum value of p – q means maximum value 4
Solution Slot – 1 (Mathematics) Page # 5

7 ⇒ – 1 ≤ sin x – 3 cos x + 1 ≤ 3
Case – 2 : a ≠
4 Range ∈ [–1, 3]
x2
f'(x) = (4a – 7 ) + 2(a – 3) x + 1 Sol.32 D
D≤0 f : R → R ; f(x) = 6x + 6|x|
(0, 2)
⇒ 4(a – 3)2 – 4(4a – 7) ≤ 0 x ≥ 0 f(x) = 2.6x
⇒ a2 – 6a + 9 – 4a + 7 ≤ 0 1 0

⇒ a2 – 10 a + 16 ≤ 0 x < 0 f(x) = 6x + 6–x = 6x + ≥2


6x
⇒ (a – 8) (a – 2) ≤ 0 ∴ 2 ≤ a ≤ 8 Many-one into
nx

Sol.29 C Sol.33 D
f : (e, ∞) → R 1
f(x) = px + sinx
f ( x )n (n (n x))
=
e x f '(x) = p + cos x
p ≠ 0 for converging ranges of f(x) is (–∞, ∞)
f '(x) = P + cos x > 0 or < 0 ∀ x ∈ R
e<x<∞ P∈ (–∞, –1) ∪ (1, ∞) → f'(x) will not be zero.
nt
Let n x = t
t ∈ (1, ∞) ⇒ n t ∈ (0, ∞) Sol.34 B
1 < nx < ∞ f : S → R+, f(∆) = area of the ∆
0 < n (n x) < ∞ O t
S → set of triangle ; R+ → set of real values
– ∞ < n (n (n x) < ∞ for one base there are many triangle can possible.
So many-one.
Range = Co-domain onto n u u ∈ (0, ∞)

Let n t = u Sol.35 C
Let nx = t ; t ∈ (1, ∞)
n t ∈ (0, ∞)
u
x2 – 4 x2 – 4
f : R → R ; f(x) = ⇒ f(– x)= = f(x)
one-one x2 + 1 x2 + 1
y
f(x) is even that's why many-one.
Sol.30 C
f(x) = 2[x] + cos x π x2 – 4
y= ⇒ yx2 + y = x2 – 4
f:R→R 2 3.14 4.72 x2 + 1
Some values are x

x=1 y+4 + – +
missing so into I ∩ S x=2 ⇒ x2 = ≥0
1– y –4 1
1.57 x=3
2 + cos 1 y+4
⇒ ≤0 ∴ y ∈ [–4, 1)
y –1
2 + cos 2
Some values are Range ≠ Co-Domain ⇒ into
1
missing so into I ∩ S
cos 1
Sol.36 B
2
= e – x +3 x – 2
2 –3 x +2 )
f :(–∞, 1) → [0, e5] ; f(x) = e –( x
0 ≤ x < 1 f(x) = cos x t(x) = – x2 + 3x – 2
1 ≤ x < 2 f(x) = 2 + cos x = – (x – 2) (x – 1)
2 ≤ x < 3 f(x) = 4 + cos x –∞ < t(x)< 0
3 ≤ x < 4 → curve will turns thats why many-one. 0 < et(x) < 1
f(x) = et(x)
Sol.31 D f(x1) = f(x2)
Range (– ∞,0]
f : R → S ; f(x) = sin x – 3 cos x + 1 t ( x1 )
=e t( x 2 )
e ⇒ t(x1) = t(x2)
– 2 ≤ sin x – x1 = x2 ⇒ t is one-one ⇒ f is one-one function.
⇒ 3 cos x ≤ 2
Page # 6 Solution Slot – 1 (Mathematics)

Sol.37 C Sol.42 D
f(g(x)) = cot–1 (2x – x2) y = f(x)
–∞ < 2x – x2 ≤ 1
But domain of f(x) is R+  1 1
f  x +  = x2 + 2 (a ≠ 0)
π π  x x
0 < 2x – x2 ≤ 1 ⇒ > cot–1 (2x – x2) ≥
2 4
2
 1  1
π π  f x +  = x +  – 2 ⇒ f(x) = x2 – 2
Range ∈  ,   x  x
4 2 

Sol.38 B Sol.43 C
f(x) = |x – 1| : f : R+ → R f(1) = 1
g(x) = ex, g : [–1, ∞) → R |ex – 1|
f(n + 1) = 2 f(n) + 1
f[g(x)] = f(ex) = |ex – 1| f(2) = f(1 +1) = 2+ 1
Range ∈ (0, ∞) O x
f(3) = 2f(2) + 1 = 2(2 + 1) + 1 = 4 + 2 + 1
Domain = g(x) = [–1, ∞)
f(4) = 2f(3) + 1 = 2 (4 + 2 + 1) + 1 = 8 + 4 + 2 + 1
f(n) = 2n–1 + 2n–2 + ........+ 8 + 4 + 2 + 1
Sol.39 B
g(x) = 1 + x – [x] f(x) = – 1 x < 0
2n – 1
0 x=0 = 1 + 2 + 4 + 8 + ... + 2n–1 = = 2n – 1
2 –1
1 x>0
⇒ f(x) = sgnx
f[g(x)] = f(1+ x – [x]) = sgn (1+ x – [x]) Sol.44 B
= sgn (1 + { x}) = 1 x2 f(x) + f(1 – x) = 2x – x4
↓ replace x → 1 – x
positive
(1 – x)2 . f(1 – x) + f(x) = 2 (1 – x) – (1 – x)4
f(x) = 1 – x2
Sol.40 A
f :[0, 1] → [1, 2] g : [1, 2] → [0, 1]
Sol.45 D
f(x) = 1 + x g (x) = 2 – x
gof(x) = g[f(x)] = g(1 + x) = 2 – (1 + x) = 1 – x f(x – y) = f(x) f(y) – f(a – x) f(a + y) (∴ f (0) = 1)
Linear polynomial thats why. put y = 0
f(x) = f(x) f (0) – f (a – x) f(a) ⇒ f(a – x) f(a) = 0...(1)
Sol.41 D put x = a, y = a – x
⇒ f(x) = f(a – x) f(a) – f(0).f(2a – x)
x 2 ; x ≥ 0
x+ | x |  ⇒ f(x) = 0 – f(2a – x) ⇒ f(2a – x) = – f(x)
f(x) = =  x ; x ≥ 0 ; g(x) = x x < 0
2 0 ; x <0
 Sol.46 D
f:R→R
 g( x ) ; x≥0 f(x + y) = f(x) + f(y)
 2
 x ; x≥0 f(1) = 7
gof = g[f(x)] = 
g(0) ; x≤0 f(2) = f(1) + f(1) = 14
 0 ; x≤0 f(3) = f(2)+ f(1) = 14 + 7 = 21
f(4) = f(2) + f(1) = 21 + 7 = 28
f ( x 2 ) ; x ≥ 0 f(5) = f(4) + f(1) = 28 + 7= 35
 2 ⇒ f(1) + f(3) + f(3) + f(4) + f(5) + ......
fog(x) = f[g(x)] =  x ; x ≥ 0
 f (x) ; x < 0 = 7 + 14 + 21 + 28 + 35 + ........
 0 ; x < 0
7n(n + 1)
= 7 (1 + 2 + 3 + 4 + 5 + .........) =
⇒ fog(x) = gof(x). 2
Solution Slot – 1 (Mathematics) Page # 7

Sol.47 B
x2 + 1
 1 + sin x  ∴ 0≤ <1
p
f(x) = log  
 1 – sin x 
5
 1 – sin x  ⇒ x2 + 1 = 5 ⇒ <1 ⇒ p>5
p
f(–x) = log  
 1 + sin x 
Sol.52 D
 1 + sin x 1 – sin x 
⇒ f(x) + f(– x) = log  × 
 1 – sin x 1 + sin x  xf ( x 2 )
f(x)= given that f(–x) = f(x) ....(1)
⇒ f(x) + f(–x) = 0 ∴ f(– x) = – f(x) odd 2 + tan 2 x.f ( x 2 )

Sol.48 D – xf ( x 2 )
f(–x) = ⇒ f(– x) = – f(x) ...(2)
2 + tan2 x + f ( x 2 )
ax – 1
f(x) = When both conditions are there only one
xn (a x + 1)
possibility is there when f(x) = 0 ⇒ f(10) = 0
1
–1 Sol.53 C
a–x –1 ax
f(– x) = = f(x) = sec (sin x)
(−x)n(a–x + 1) 1 
(– x )n  + 1 f(T) = f(0) n=1
a  sec (sinT) = 1 T=π
sin T= 0 ⇒ T = nπ
–(a x – 1)
= Sol.54 D
(a x + 1)(–x)n

⇒ f(– x) = f(x) ⇒ – (– x) = 1 ⇒ n = –
n
1 f(x) = sin [a ] x ⇒ =π ⇒ 4 = [a]
[a]
3
∴ a ∈ [4, 5)
Sol.49 B
even f(– x) = f(x) Sol.55 A
Even function are f(x) = x + a – [x + b] + sinπx + cos2π x + sin 3πx
t
symmetric about + cos 4πx +...+ sin (2n – 1) πx + cos 2nπx
x=0
y-axis that means f(2–t)
f(x) = x + a – (x + b) + {x + b} + sin πx + cos 2πx
symmetric about
f(2+t)
+ sin 3πx + cos 4πx + ... + sin(2n – 1) πx
–t +t
x = 0 line. + cos 2nπx
x=2
f(2 – x) = f(2 + x) sin πx cos 2πx sin 3πx
2π 2 π 2 π 2π 2 π 2π 2π
Sol.50 B Period , , , , ... ,
π 2π 3 π 4π 5 x (2n – 1)π 2nπ
f(–x) = – f(x)
By Definition
2 2 2 2 2  2   2 
f(x) is an odd function , , , ....  ,  
1 2 3 4 5  2n – 1   2n 
Sol.51 C 2
Period = =2
 x 2 + 1 1
g : [–2, 2] → R ; g(x) = x + tan x +  p 
3
 
Sol.56 C

 x 2 + 1 π πx π
f(x) = sin [x] + cos + cos [x]
g(– x) = – g(x) ⇒  p  = 0 4 2 3
  If 0 ≤ x < 1 then y = 0
Page # 8 Solution Slot – 1 (Mathematics)

1
If 1 ≤ x < 2 then y = Sol.58 A
2
If 2 ≤ x < 3 then y = 1 16 x – 1 1
(A) f(x)= = 4x –
4 x 4x
1
If 3 ≤ x < 4 then y =
2 1 1  x 1 
f(– x) = 4–x – – x = x – 4x = –  4 – x  = – f(x)
If 4 ≤ x < 5 then y = 0 4 4  4 
f(–x) = – f(x) odd
(B) f(x) = sin |x|
2π f(– x) = sin |– x| = sin |x| = f(x)
=8
π 0 1 2 3 4 5 6 7 8 9
 
4 Sol.59 C
f(x) = x (2 – x) ; f(x + 2) = f(x) ⇒ period = 2
Period 8/4/6 ⇒ Lcm = 24
Sol.60 C
f(2, 4) → (1, 3)
Sol.57 A
x x
 1  2 2 < x< 4 ; 1 < <2 ⇒   =1
f(x) = [x] +  x + + x + 2
3  3 
– 3x+ 15 2
 
y = f(x) = x – 1 ⇒ x= y+ 1
 1  1  2  1 f–1 (y) = y + 1 ; f–1 (x) = x + 1
f  x +  =  x +  +  x +  +[x + 1] – 3  x +  +15
 3  3  3  3
Sol.61 B
 1  1  2 f : R → R ; f(x) = x3 + ax2 + bx + c
f  x +  = x + + x+
3   3 
+[x]+1 – 3x – 1 +15 f ’(x) = 3x2 + 2ax + b > 0
 3 
D ≤ 0 ⇒ 4a2 – 4(3)b ≤ 0
 1  1  2 ⇒ a2 – 3b ≤ 0 ⇒ a2 ≤ 3b
f  x +  = [x] +  x + 3  +  x + 3  – 3x + 15
 3    

 1 1
f  x +  = f(x) ⇒ period =
 3  3

EXERCISE – II HINTS & SOLUTIONS


Sol.1 A,B
2 2x
f : [– 1, 1] → [0, 2] (B) y= [sin–1 (sin x)] =
π π
y=1–x one-one - into
y=0 y=1+x Range ∉ co-domain
–1 1
Not a Bijective function
x =1 y=2 (C) y=x 0<x≤ 1
y=2 x = –1 y=0
= – x –1 ≤ x < 0
Range = Co-domain =0 x= 0
Not a Bijective function
Sol.2 B,C,D (D) y = x3 0<x≤ 1
–1
(A) y = sin (sin x) = x (Bijective function) = – x –1 ≤ x < 0
3

=0 x= 0
Not a Bijective function
Solution Slot – 1 (Mathematics) Page # 9

Sol.3 A,C Sol.8 B,C,D


f:N→I
sin π[ x]
0 1 2 3 f(x) = =0 Range = 0
{ x}
n–1
f(x) =
2
n is odd n = 1, 3, 5,7........ x ∉ I ⇒ x ∈ R – (I) f(x) = 0 ∀ x ∈ R – I

n
=– n is even n = 2, 4, 6, 8........
2
–1 –2 –3 –4 –4 –3 –2 –1 1 2 3 4
one-one onto.

Sol.4 A,C
  { x}  
1– x g(x) = sgn  sgn  – 1 = 1 – 1 = 0 ; x ∈ R – I
f(x) = ; g(x) = 4x(1 – x)   {x}  
1+ x   

g(x) = 0 ∀ x ∈ R – I
1 – 4 x(1 – x ) 1 – 4 x + 4 x 2
f[g(x)] = f (4x (1 – x))= =
1 + 4 x(1 – x ) 1 + 4 x – 4 x 2
Sol.9 A,D

 1– x   1 – x   1 – x  8 x(1 – x ) f(x) = sinx + tanx + sgn (x2 – 6x + 10) = 1


g[f(x)] = g  = 4  1 – =
⇓ ⇓ ⇓
 1+ x   1+ x   1 + x  (1 + x )2
2π π D = 36 – 40 < 0
2π 2π
Sol.5 A,B,C
3π 3π
By definition
4π 4π

Sol.6 A,B,C
Sol.10 A,B,C,D
f(x) = sin4 3x + cos4 3x
f(T) = f(0) (A) f(x) = sgn e–x
⇒ sin43T + cos43T = 1 (B) f(x) = 1 ; x is Rational
⇒ sin43T = 1 – cos43T = sin23T (1 + cos23T) = 0 ; x is Irrational
⇒ sin23T (sin23T – 1 – cos23T) = 0
8 8 4
∴ sin23T = 0 or cos23T – sin23T = – 1 (C) f(x) = + =
1 + cos 1 – cos x | sin x|
3T= nπ or cos 6T = – 1 ⇒ 6T = 2nπ ± π

nπ π period = π
T= or T = (2n + 1)
3 6
 1  1
(D) f(x) =  x +  + x –  + 2 [– x]
π π 3π π  2  2
n=1 T= or n = 0 T = ; n =1 T = =
3 6 6 2
 1  1  1  1
=  x +  – x +  +  x –  + x − 2  +2 (– x) – 2 {– x}
Sol.7 B,C,D  2   2  2  
f : R → [–1, 1]
0≤x<1; y=0  1  1
= 2x – 2x –  x +  + x –  – 2 {–x}
1
1≤x<2; y=1  2  2
2≤x<3; y=0 0 1 2 3 4 5 6

3 ≤ x < 4 ; y = –1 –1
 1  1
4≤x<5; y=0 = –  x +  + x –  – 2 {– x}
 2  2
5≤x<6; y=1
Page # 10 Solution Slot – 1 (Mathematics)

EXERCISE – III HINTS & SOLUTIONS

∴ x ∈ [–1, 1]
x3 – 5x + 3
Sol.1 (i) f(x) = 1
x2 – 1 (viii) f(x) = (x2 + x + 1)–3/2 =
x2 – 1 ≠ 0 ⇒ x ≠ ± 1 ∴ x ∈ R – {– 1, 1} ( x + x + 1)3 / 2
2

∴ x∈ R
1
(ii) f(x) = x+ | x | x–2 1– x
(ix) f(x) = +
x+2 1+ x
x + |x| > 0 ⇒ x > – |x| ∴ x ∈ (0, ∞)
(iii) f(x) = ex + sinx ∴ x∈R x–2 + – +
≥0
1 x+2 –2 2
(iv) f(x) = log (1 – x ) + x+2
10
∴ x ∈ (–∞, – 2) ∪ [2, ∞)
log10 (1 – x) ≠ 0 ⇒ 1 – x ≠ 1 ⇒x≠ 0
&1–x≠ 0 ⇒x≠ –1 1– x x –1
& ≥0 ⇒ ≤ 0 ⇒ –1 < x ≤ 1
&1 –x>0 ⇒x< 1 1+ x x +1
& x + 2 ≥ 0 ⇒ x ≥ – 2 –2 ∴x∈φ
∴ x ∈ [–2, 0) ∪ (0, 1)
(x) tan x – tan 2 x
f(x) =
    ⇒ tanx – tan2 x ≥ 0 ⇒ tan2x – tan x ≤ 0
 1   1 
(v) logx log2   ⇒ log2   > 0;x≠1 ⇒ tan x (tan x – 1) ≤ 0
x– 1 x– 1
     π
 2  2 0 ≤ tan x ≤ 1 ; x ∈ 0, 
 4
1 1
⇒ > 1 ; x< 0 ⇒ –1 > 0
1 1 π  π
x– x– 0≤ x≤ ; x ∈ nπ,nπ + 
2 2 4  4

1 3 1
1– x + x– (xi) f(x) =
2 >0 ⇒ 2 <0 1 – cos x

1 1
x+ x– 1 – cos x > 0 cos x ≠ 1
2 2 cos x < 1 x ≠ 2nπ
∴ x ∈ R – {2nπ}
1  3 
∴ x ∈  ,1 ∪  ,1
2  2   5x – x 2 
(xii) f(x) = log1/4  
 4 
 
(vi) f(x) = 3 – 2 x – 21– x

2  5x – x2 
⇒ 3–2x – 21–x ≥ 0 ⇒ 3 ≥ 2x + (Let t = 2x)  
2 x log1/4  4  ≥0
 

t2 + 2
⇒ 3≥ ⇒ t2 + 2 ≤ 3t 5x – x2 x 2 – 5x + 4
2t ⇒ ≤1⇒ ≥0
4 4
⇒ t2 + 2 < 3t ⇒ t2 – 3t + 2 ≤ 0
⇒ 1≤t≤2 ⇒ 1 ≤ 2x ≤ 2 ⇒ (x – 4) (x – 1) ≥ 0 ∴x≥ 4; x≤1
∴ 0 ≤ x≤1
5x – x2
(vii) f(x) = 2 & > 0 ⇒ x (x – 5) < 0 ⇒ 0 < x< 5
1– 1– x 4

1 – 1− x 2 ≥ 0 ⇒ 1 ≥ 1− x 2 ∴ x ∈ (0, 1] ∪ [4, 5)
0 1 4 5
⇒1–x ≤1 2
⇒ –x2 ≤ 0 ⇒ x2 ≥ 0 ∴ x ∈ R
& 1 – x ≥ 0 ⇒ x2 – 1 ≤ 0 ⇒ –1 ≤ x ≤ 1
2
Solution Slot – 1 (Mathematics) Page # 11

(xiii) f(x) = log10 (1 – log10 (x2 – 5x + 16)) x ≥ 4 y = 1 ; x < 4 y = – 1 ⇒ y ∈ {–1, 1}


1 – log10 (x2 – 5x +16) > 0 (v) f(x) = 5 + 3 sinx + 4 cos x
⇒ log10 (x2 – 5x + 16) < 1 ⇒ x2 – 5x + 16 < 10 – 5 ≤ 3 sin x + 4 cos x ≤ 5
⇒ x2 – 5x + 6 < 0 ∴ 2<x<3 ⇒ 0 ≤ 5 + 3 sin x + 4 cos x ≤ 10 ⇒ 0 ≤ y ≤ 10
2
& x – 5x + 6 > 0 ⇒ x∈R
1
Domain x ∈ (2, 3). (vi) f(x) = =y
1+ x
Sol.2 (i) f(x) = |x – 3| is always positive ∴ y ∈ [0, ∞)
1– y y –1
x x = y ≥ 0 ⇒ y ≤ 0 ⇒ y ∈ (0, 1]
(ii) f(x) = =y
1+ x2 (vii) y = 2 – 3x + 5x2 = – (5x2 + 3x – 2)
⇒ x = y + yx2 ⇒ yx2 – x + yx2
 –D  (9 + 40) 49  49 
1 1 –   = = ∴ y ∈  – ∞, 
D≥0 ⇒ 4y2 – 1 ≤ 0 ⇒ – ≤ y ≤  4a  20 20  20 
2 2
(viii) y = 3 |sin x| – 4 |cos x|
x ↑ ↑
Aliter (1) : y = ymin = min max
1+ x2
ymin = 0 – 4 = – 4
Df x ∈ R ymax = 3(1) – 4(0) = 3 ∴ y ∈ [–4, 3]
sin cos x
x+
1
≥2 cos x ≠ 0
(ix) y = +
x=0 x≠0 x 1 + tan 2 x 1 + cot 2 x sin x ≠ 0
y=0 1
y= 1
x + ≤ –2
1 sin x cos x
x+ x
x = | sec x | +
| cos ecx |
1 1 1 = sin x |cos x| + |cos x| sin x|
⇒ x+ ≥2 ⇒ ≤
x 1 2 cos x & sin x can't be on vertical & Horizontal line.
x+
x π
Case – I : 0 < x < ⇒ 0 < 2x < π
2
1 1 1  1 1
⇒ – ≤ ≤ ⇒ y∈ – 2 , 2  f(x) = sin 2x ∈ (0, 1]
2 1 2  
x+
x π 
Case – II : x ∈  , π 
2 
x
Aliter (2) : y = ; x∈ R f(x) = – sin x cos x + sin x cos x = 0
1+ x2

Case – III : π < x < ⇒ 2π < 2x < 3π
1 2
x = tan θ ⇒ y = sin 2θ =
2 f(x) = – sinx cos x – sinx cos x = – sin 2x
0 < sin 2x ≤ 1 ⇒ – 1 ≤ – sin 2x < 0
 1 1
– 2 , 2  . ∴ f(x) ∈ [–1,0)
 

(0, 4) Case – IV : < x < 2π
(iii) f(x) = 16 – x 2 2
f(x) = 0 ∴ y ∈ [–1, 1]
y= 16 – x 2 (x) f(x) = 1 – |x – 2| ⇒ y = 1 – |x – 2|
⇒ |x – 2| = 1 – y ≥ 0 ∴ y≤1
y ∈ [0, 4] (–4, 0) (–1, 0)

Half part of circle 1 1


x2 = 16 – y2 ≥ 0 (0, –4) (xi) y=
x–5
⇒ x – 5 = y >0 ∴ y ∈ R+
⇒ y2 – 16 ≤ 0
– 4 ≤ y ≤ 4 But y can take only +ve value 1 1
0≤y≤ 4 (xii) f(x) = = y ⇒ y = 2 – cos 3x
2 – cos 3 x
|x–4|
(iv) f(x) =
x–4
Page # 12 Solution Slot – 1 (Mathematics)

= sin2x + 1 + sin4x – 2 sin2x = 1 + sin4x – sin2x


1
⇒ cos 3x = 2 – y 2
 2 1 3
=  sin x –  + ; sin2x = 0, y = 1
 2  4
1 1
⇒ –1≤2– y ≤1 ⇒ –3≤– y ≤–1
3 3 
min value = ; sin2x = 1, y = 1 ∴  , 1
4 4 
1 1
⇒ 1≤ y ≤3 ⇒ ≤ y ≤1
3
1 1
x+2 Sol.3 (i) f(x) = ⇒ y=
(xiii) y = 2 ⇒ yx – 8xy – 4y – x – 2 = 0
2 4 + 3 sin x 4 + 3 sin x
x – 8x – 4
⇒ yx2 – x (8y + 1) – (4y + 2) = 0 4
4 + 3 sin x > 0 ⇒ sin x > – ∴ x∈R
D ≥ 0 ⇒ (8y + 1)2 + 4y (4y + 2) ≥ 0 3
⇒ 80y2 + 24 y + 1 ≥ 0
⇒ (4y+ 1) (20 y + 1) ≥ 0 1 1
& y2 = ⇒ 3 sinx = 2 – 4
4 + 3 sin x y
 1  1 
∴ y ∈  – ∞, –  ∪  – ,∞ 
 4  20  1 1 1
⇒1≤ ≤7⇒ ≤ y2 ≤ 1 ⇒ ≤ y ≤1
y2 7 7
x 2 – 2x + 4
(xiv) y= (ii) f(x) = x!
x 2 + 2x + 4 Domain = N ∪ {0}
⇒ yx2 + 2xy + 4y = x2 – 2x + 4 Range = n!, {n = 0, 1, 2, 3, .........}
⇒ (y – 1) x2 + 2π (y + 1) + 4(y – 1 ) = 0
at y = 1 ⇒ 2x (2) = 0 ⇒ x = 0 x2 – 9
(iii) f(x) =
D ≥ 0 ⇒ 4(y+ 1)2 – 16 (y – 1)2 ≥ 0 x–3
1 y=x+3x≠ 3
⇒ (y – 3) (3y – 1) ≤ 0 ∴ ≤y≤3 Domain R – {3} ; Range R – {6}
3
(iv) f(x) = sin2(x3) + cos2(x3)
Domain x ∈ R ; Range = {1}
π2
(xv) f(x) = 3 sin – x2
16
4x
π 2
π 2 Sol.4 f(x) =
0≤ – x2 ≤ 4x + 2
16 16
41– x 4 2
π 2 π π f(1 – x) = = x =
⇒ 0≤ – x2 ≤ ⇒ 0≤θ≤ 41– x
+2 4 + 2 .4 2 + 4x
16 4 4
4x 2
1 3 ⇒ f(x) + f(1 – x) = + =1
⇒ 0 ≤ sin θ ≤ ⇒ 0 < 3 sin θ < 4 +2x 4 + 2x
2 2
(xvi) f(x) = x4 – 2x2 + 5 = (x2 – 1)2 + 4 Sol.5 f(x) = |x2 – 4| x| + 3|
min at x = 1, min. value = 4 (0, 3)
range = [4, ∞)
Aliter
x2 = – t ≥ 0
4 [4,∞ )
t2 – 2t + 5
–3 2 –1 1 3
 b –D 
 – 2a , 4a  = (1, 4) ⇒ a ∈ (1, 3) ∪ {1}
 
(xvii) f(x) = x3 – 12 x x ∈ [–3, 1] Sol.6 2x + 3 [x] – 4 {–x} = 4
at x = – 3 ; y = – 27 + 3 = 9 Case–I : x ∈ I
at x = 1 ; y = 1 – 12 = – 11 ∴ y ∈ [–11, 9]
(xviii) f(x) = sin2x + cos4x = sin2x + (1 – sin2x)2
Solution Slot – 1 (Mathematics) Page # 13

4
2x + 3 x – 0 = 4 ⇒ x = (reject) Sol.9 (i) f(x) = |x2 + 5x + 6| = |(x+3) (x+2)| = Many one
5
Case–II : x ∉ I
2x + 3 [x] – 4 (1 – {x}) = 4
2x + 3 [x] – 4 + 4 {x} = 4
2x + 3 [x] – 4 + 4 (x – [x]) = 4 –3 –2 2 3
2x + 3 [x] – 4 + 4x – 4 [x] = 4
6x = 8 + [ x] ...(1)
|log x|
2x + 3 (x – {x}) – 4 + 4{x} = 4
2x + 3x – 3{x} – 4 + 4 {x} = 4
5x + {x} = 8
0 ≤ {x} < 1 ⇒ 0 ≤ 8 – 5x < 1 (ii) |log x| ⇒ Many-one.
7 8
⇒ – 8 ≤ – 5x < – 7 ⇒ <x≤
5 5 (iii) f(x) = sin 4x
9 3 π
[x] = 1 ⇒ 6x = 8 + 1 ⇒ x = = .  – π π –
6 2 x∈  ,  8
 8 8
π
one-one 8
Sol.7 f(x) → [– 2, 2]
1
x ∈ (0, ∞)
(0, 1)
(iv) f(x) = x +
x
 −1 ; − 2 ≤ x ≤ 0 (1, 0)(2, 0) f(x) ≥ 2
f(x) = x − 1 ; 0 ≤ x ≤ 2 –2 –1


(0, –1)
1
f '(x) = 1 – =0
x2
(0, 1)
x2 = 1 ⇒x=±1
 1 ; −2≤ x ≤0
|f(x)| =  − x + 1 ; 0 ≤ x ≤1 x= 1 ⇒ f '(x) = 0
–2 –1 (1, 0)
 x − 1 ; − 1 ≤ x ≤ 2 Curve turned because of f'(x) = 0
So many-one.
1
x → 0+ f(x) → 0+ + →∞
0+ 2
− x − 1 ; − 2 ≤ x ≤ 0

f(|x|) =  x − 1 ; 0 ≤ x ≤ 1 (–2, 0) (–1, 0) (1, 0) (2, 0)
1
 x − 1 ; 1 ≤ x ≤ 2 x → ∞ f(x) → ∞ + =∞
(0, – 1) ∞
Aliter :

 − x ; −2≤ x ≤0
x = 2 ; f(2) = 2 +
1
=
5
∴ f(|x|) + |f(x)| =  0 ; 0≤ x ≤1 2 2
 2(x − 2) ; 1≤ x < 2
1  1 1 5
x= ; f  = +2=
2  
2 2 2
2
Sol.8 (i) f(x) = x 2 & g(x) = ( x )
Domain = x ∈ R ; x ∈ R+ ∪ {0}  1
f(2) & f   are same so many one.
(ii) f(x) = sec (sec x) g(x) = cosec (cosec–1 x)
–1
2
Domain are same
1 
1+ cos 2x (v) f(x) =  –1
(iii) f(x) = ; g(x) = cos x 1 – e x 
2
f(x) = |cos x| ; g(x) = cos x 1 
 –1 1
Appearance is not same. 1– e x  ≥0 ⇒ –1≤0
x
(iv) f(x) = x & g(x) = enx
Domain = x ∈ R Domain = x ∈ R+
Page # 14 Solution Slot – 1 (Mathematics)

1– x x –1
⇒ ≤0 ⇒ ≥0 Sol.11 (i) f(x) = x|x|
x x
x ≥ 0 f(x) = x2
∴ x ∈ (–∞, 0) ∪ [1, ∞) x < 0 f(x) = – x2
Range = co-domain Range = R
f(1) = 1 – e1–1 = 0
one-one, onto
1 (ii) f(x) = x2
Lt –1 =1
x →0 −
1– e x many one, into

x2
Lt 1
–1 1–
1 (iii) y= ≥ 0 always
x→∞ 1– e x
=
e 1 + x2
f(x) = f(–x) ⇒ many-one, into
Lt 1
–1 1 (iv) f(x) = x3 – 6x2 + 11 x – 6
= 1–
x →– ∞ 1– e x e f '(x) = 3x2 – 12 x + 11
D ≥ 0, Range = co-domain ⇒ onto
1
1
1– Sol.12 (i) x – sin x
e
f ’(x) = 1 – cos x ≥ 0 ⇒ many one.
(ii) y = x |x|
x ≥ 0 y = x2

x < 0 y = – x2
f is one-one. ⇒ one-one & onto
πx
3x 2 (iii) tan
(vi) f(x) = – cos πx 4

– 1.57
π
2 x = 1 ; y = tan =1
3x 1.57
f(– x) = – cos πx = f(x) 4
4π x= –1 ;y=– 1
even function so many-one ⇒ one-one-onto
(iv) y = x4 ⇒ many-one
6
1+ x
Sol.10 (i) f(x) = Sol.13 (i) f(x) = ex ; g(x) = log x
x3 2
fog(x) = f[g(x)] = f[logx] = elog x
Domain x ∈ R – {0} gof(x) = g[f(x)] = g(ex) = log ex = x log e
(ii) fog(x) = f[g(x)] = f[sin x] = |sin x|
1 –2
3
f(x) = x + gof(x) = g[f(x)] = g(|x|) = sin |x|
x3 (iii) fog(x) = f[g(x)] = f[x2] = sin–1 x2
If x ≥ 0 f(x) ≥ 2 ; x < 0 f(x) < – 2 gof(x) = g[f(x)] = g(sin–1 x) = (sin–1 x2)2
Range ≠ co-domain ⇒ Into function.
(ii) f(x) = x cos x  1  3x 2 − 4 x + 2
(iv) fog(x) = f[g(x)] = f 1 − =
↓ ↓  1− x  (1 − x )2
x ∈ R ; [–1, 1]
Range = co-domain ⇒ onto x2 + 2
gof(x) = g[f(x)] = g(x2 + 2) =
1 x2 + 1
(iii) f(x) = sin | x |
Sol.14 f(x) = 1 + x2 ; x ≤ 1 3
x ∈ R – {0} ; y ∈ R – {0} 2
=x+1;1<x≤2
Range ≠ Co-domain ⇒ Into 1
g(x) = 1 – x ; – 2 ≤ x ≤ 1
(iv) tan(2 sin x) –2 –1
0
1
– 1 ≤ sin x ≤ 1
f[g(x)] = 1 + g(x)] ; g (x) ≤ 1
2
–2 ≤ 2 sin x ≤ 2

π π
= g(x) + 1 ; 1 < g(x) ≤ 2
– tan 2 ≤ (2 sin x) ≤ tan 2 2 2
–1.57 1.57 fog(x) = f[g(x)] = 1 + [g(x)]2 ; g(x) ≤ 1
– 0.035 0.035
Solution Slot – 1 (Mathematics) Page # 15

= g(x) + 1 ; 1 < g(x) ≤ 2 (1 + 2– x )7


f[g(x)] = 1 + [g(x)]2 = x2 – 2x + 2 ; 0 ≤ x ≤ 1 f(– x) = none
= g(x) + 1 = 2 – x ; – 1 ≤ x < 0
2– x
sec x + x 2 – 9
Sol.15 f(x) = 1 + x ; 0 ≤ x ≤ 2 3 (ii) f(x) =
x sin x
=3– x ; 2< x≤ 3 2
1+x
1 3–x sec x + x 2 – 9
f[f(x)] = 1 + f(x) ; 0 ≤ f(x) ≤ 2 f(–x) = even
x sin x
= 3 – f(x) ; 2 < f(x) ≤ 3 1 2 3
= 1+ 1+ x= 2+ x; 0≤ x≤ 1 (iii) f(x) = 1 + x + x2 – 1 – x + x2
=4–x ;2< x≤ 3
= 3 – (1 + x) = 2– x ; 1 < x ≤ 2 f(– x) = 1 – x + x2 – 1 + x + x2
f(– x) = – f(x) odd
Sol.16 f(x) = n (x2 – x + 2) : R+ → R (iv) f(x) = x|x| ; x ≤ – 1
g(x) = {x} + 1 : [1, 2] → [1, 2] = [1+ x] – [x – 1] = 2 ; – 1 < x < 1
f(g(x)) = f({x} + 1) a = {x} = n [(a + 1)2 – (a + 1) + 2] = – x |x| ; x ≥ 1
= n (a2 + 2a + 1 – a – 1 + 2) = n (a2 + a + 2) f(– x) = – x|x| x ≥ 1
domain of f(g(x)) = domain of g(x) = [1, 2] =2;–1<x<1
0 ≤ {x} < 1 ⇒ 0 ≤ {x}2 < 1 = x |x| ; x ≤ –1. even
⇒ 0 ≤ {x} + {x} < 2
2
⇒ 2 ≤ a2 + a + 2 < 4
2x (sin x + tan x ) 2x (sin x + tan x )
⇒ n 2 ≤ n(a2 + a + 2) < n4 (v) f(x) + f(–x) = +  x
 x
∴ Range ∈ [n2, n4] 2 2 +  − 3 2 2 −  − 3
 π  π

 
 1  1  
Sol.17 f(x).f   = f(x) + f    1
+
1 
x x = 2x(sin x + tan x)  x 
 2   + 1 2 − x  + 1
 
  π   
f(x) = 1 ± xn  π  
f(3) = 1 ± (3)n = – 26
  x  − x  
1 + 3 n = −26 ⇒ 3 n = −27  2    +    + 2 
 π π 
⇒  = 2x(sin x + tan x)        
1 − 3n = −26 ⇒ 3n = 27 ⇒ n = 3   2  x  + 1  2  − x  + 1 
   π     π   
 
f(x) = 1 – x3 ⇒ f’(x) = – 3x2 ⇒ f’(1) = – 3
 x
Sol.18 f(x + y) = f(x).f(y) ; f(1) = 2 x − x  0 ; π ∈ n or x ∈ n π
f(x) = ax ⇒ f(1) = a = 2 ⇒ f(x) = 2x π +  π  =  x
    − 1 ; ∉ n or x ∉ n π
 π
10
∑ f(n) = 2 + 2 + 2 2 3
+ .......... + 210 Case – I : x = nπ
n =1 f(x) + f(–x) = 0 ⇒ f(–x) = – f(x)
Case – II : x ≠ nπ
 210 – 1  f(x) + f(–x) = 0 ⇒ f(–x) = – f(x)
 
= 2  2 – 1  = (210 – 1)2 = 2 (1023) = 2046 f(x) is an odd function
 

Sol.19 f(x) = x2 + sin x ; 0 ≤ x < 1 Sol.21 (i) f(x) = 2 + 3 cos (x – 2)


= x + e–x ; x ≥ 1 Period = 2π or f(T) = f(0)
(i) An even function : f(–x) = f(x) (ii) f(x) = sin3x + cos2 x + |tan x|
f(x) = x2 – sin x ; – 1 < x ≤ 0 1+ cos 2x
= – x + ex ; x ≤ – 1 = sin 3x + + |tan x |
2
(ii) odd function : f(x) = – f(–x)
f(x) = – x2 + sin x ; – 1 < x ≤ 0 ↓ ↓ ↓
= x – ex ;x≤– 1 2π 2π
= π
3 2
(1 + 2 x )7 LCM 2π
Sol.20 (i) f(x) = = = 2π
HCF 1
2x
Page # 16 Solution Slot – 1 (Mathematics)

1
πx πx = 1 – 1 + sin x cos x = sin 2x
(iii) f(x) = sin + sin 2
4 3

↓ ↓ ∴ = π period
2
2π 2π
=8 =6 (ii) f(x) = log (2 + cos 3x)
π/4 π/3
f(T) = f(0)
LCM 8×3
= = 24 ⇒ log (2 + cos 3T) = log 3 ⇒ 2 + cos T = 3
HCF 1
⇒ cos T = 1 ⇒ 3T =2π ⇒ T = 2π/3
3 2
(iv) f(x) = cos x – sin x  2π 
5 7 ∴ fx + = f(x)
 3 
↓ ↓
2π 10π 2π π
= = 7π (iii) f(x) = tan [π]
3/5 3 2/7 2

LCM 70π π
= = 70π f(T) = f(0) ⇒ tan [T] = 0
HCF 1 2
(v) f(x) = [sin 3x] + |cos 6x|
π
2π π ⇒ [T] = π ⇒ [T] = 2 ⇒ T = 2
2
period of [sin 3x] = ; period of |cos x| =
3 6 (iv) f(x) = ensinx + tan3x – cosec(3x – 5)
2π = sinx + tan3x – cosec (3x – 5)
∴ period of f(x) = ↓ ↓ ↓
3
2π π 2π/3
1 1 LCM 2π
(vi) f(x) = = x = = 2π
1 − cos x 2 sin2 HCF 1
2

 
π 1  | sin x | sin x 
period of f(x) = = 2π (v) f(x) =  + 
(1/ 2) 2  cos x 1 
 
 2 
sin12 x sin12 x
(vii) f(x) = 2 = period of |sin x| = π
1 + cos 6 x 1 1
+ cos 12 x
3 2 period of sin x = 2π
period of cos x = 2π
2π π period = 2π
Period of sin 12x = =
12 6
x x x
(vi) f(x) = sin x + tan + sin 2 + tan 3
2π π π 2 2 2
Period of cos 12x = = ∴ Period =
12 6 6 ↓ ↓ ↓ ↓
Period 2π π 23π 23π
1 1
(viii) f(x) = sec2x + cosec3x = + x x
2
cos x sin3 x + ........... + sin n–1 + tan
2 2n
↓ ↓ ↓ ↓
π 2π 2n π 2n π
Period = 2π LCM of (2π, 2π, 23π, 23π ........ 2nπ, 2nπ) = 2nπ
sin x + sin 3 x 2 sin 2x cos x
sin 2 x cos2 x (vii) f(x) = =
Sol.22 (i) f(x) = 1 – – cos x + cos 3 x 2 cos 2x cos x
1 + cot x 1 + tan x period = π
Solution Slot – 1 (Mathematics) Page # 17

Sol.23 (i) f(x + p) = 1 + {1 – 3f(x) + 3f2(x) – f3(x)}1/3


 π
(f(x + p) – 1)3 = 1 – 3f(x) + 3f2(x) – f3(x) ⇒ – 2 ≤ 2 sin  2π +  ≤ 2 ⇒ 0 ≤ f(x) ≤ 4 ; B ∈ [0, 4]
(1 – f(x + p))3 = (f(x) – 1)3  6
replace x → x + p
 π
(1 – f(x + 2p))3 = (f(x + p) – 1)3 ∴ 2 sin  2x +  = y – 2
(1 – f(x + 2p))3 = – (f(x) – 1)3 = (1 – f(x))3  6 
f(x + 2p) = f(x) period = 2p
(ii) f(x – 1) + f(x + 3) = f(x + 1) + f(x + 5)....(1)  π 1
⇒ sin  2x +  = (y – 2)
Replace x by x + 2  6 2
f(x + 1) + f(x + 5) = f(x + 3) + f(x + 7) ....(2)
Add (1) and (2) we get f(x – 1) = f(x + 7) π  y – 2
⇒ 2x + = sin–1  
replace x → x + 1 6  2 
f(x) = f(x + 8) period = 8
–1 1 y–2 π
⇒ f (y) = x = sin –1  –
2  2  12
e 2 x – e –2 x
Sol.24 f : R → R ; f(x) =
2
Sol.26 f : N → N ; f(x) = x + (– 1)x–1
e 2 x – e –2 x Case–I : x = 2K ; K ∈ N
f(x) = ≤ –1 one-one onto f(x) = x – 1 ⇒ y=x–1
2
f (x) = x + 1 ⇒ f (x) = x – (– 1)x–1
–1 –1

Case–II : x = 2K + 1 ; K ∈ N
2e 2 x (–2) –2x
f'(x) = – e = e2x + e–2x > 0 one-one onto. f(x) = x + 1 ⇒ y= x+1 ⇒ x= y– 1
2 2
⇒ f–1(x) = x – 1 ⇒ f–1 (x) = x – (–1)x – 1
e 2 x – e –2 x 1
y= ⇒ 2y = t –
2 t

2y ± 4y2 + 4y
⇒ t2 – 2y t – 1 = 0 ⇒ t=
2

∴ e2x = y ± y 2 + 1 ⇒ 2x = n (y ± y2 + 1 )

1
⇒ x= n (y + y2 + 1 )
2

1
∴ f −1(x) = n(x + x 2 + 1)
2

 π π
Sol.25 f :  – ,  → B
 3 6

f(x) = 2 cos2 x + 3 sin2x + 1

 π
=1 + cos 2x + 3 sin 2x + 1 = 2 + 2 sin  2x + 
 6

 π
∵ – 1 ≤ sin  2x +  ≤ 1
 6
Page # 18 Solution Slot – 1 (Mathematics)

EXERCISE – IV HINTS & SOLUTIONS

Sol.1 (i) f(x) = cos 2x + 16 – x 2 1 – 5x


(iv) f(x) =
⇒ 16 – x ≥ 0 ⇒ x – 16 ≤ 0 ⇒ – 4 ≤ x ≤ 4.
2 2
7–x – 7
& cos 2x ≥ 0
1 – 5x 1 – 5x 5x – 1
≥0⇒ ≥ 0 ⇒ ≥0
7– x – 7 1 – 71+ x 71+ x – 1
– 3π / 4 –π / 2 π/2 5x – 1 = 0 ⇒ x = 0 & 7x+1 = 1 ⇒ x = –1
–π π
5π – O π 3π π 5π
+ – +
– 4 4 4 4
4
–1 0
x=–4 x=4

 5 π 3π   π π  3π 5π 
(v) y = log10 sin (x – 3) + 16 – x 2
x ∈ – ,–  ∪  – ,  ∪  ,  16 – x2 ≥ 0 ⇒ x2 – 16 ≤ 0 ⇒ – 4 ≤ x ≤ 4
 4 4  4 4 4 4 
& sin (x – 3) > 0
(ii) f(x) = log7 log5 log2 (2x3 + 5x2 – 14x)
⇒ log5 log3 log2 (2x3 + 5x2 – 14x) > 0
⇒ log3 log2 (2x3 + 5x2 – 14 x) > 1 3 – 2π 3–π
⇒ log2 (2x3 + 5x2 – 14x) > 3
⇒ 2x3 + 5x2 – 14x – 8 > 0 O 3

⇒ (x – 2) (2x + 1) (x + 4) > 0
– + – +
x=–4 x=4
–4 –1/2 –2 ∴ x ∈ (3 – 2π, 3 – π) ∪ (3, 4]
 1 1   2 log10 x + 1 
∴ x ∈  – 4,–  ∪  , ∞  (vi) f(x) = log100x  
 2 2   –x 
100 x > 0 ⇒ x > 0
(iii) f(x) = n ( x 2 – 5 x – 24 – x – 2)
1
2 100 x ≠ 1 ⇒ x ≠
x – 5 x – 24 > x + 2 100
& x2 – 5x – 24 ≥ 0 ⇒ (x – 8) (x + 3) ≥ 0 2log10 x + 1 + – +
⇒ x≥ 8;x≤– 3 >0
–x 0 1
Case–1 : x + 2 < 0 ⇒ x < – 2
10
2 log10 x + 1  1 
x 2 – 5 x – 24 > x + 2 ⇒ <0 ∴ x∈  0, 
x  10 
+ve +ve ⇒ x < –2
Case–2 : x + 2 ≥ 0 ⇒ x ≥ –2 2 log10 x + 1
Aliter : >0 ; x> 0
–x
x 2 – 5 x – 24 > (x + 2)
x2 – 5x – 24 > x2 + 4x + 4 1
⇒ 2 log10 x + 1 < 0 ⇒ x <
10
28
x<–
9 28 –2
–  1   1 
9 ∴ x ∈  0,  –  
x ∈φ  10  100 
Case–I ∪ Case–II
x < –2
1
x < – 2 ∩ (–∞, – 3] ∪ [8, ∞) ∴ x ∈ (–∞, –3] (vii) f(x) = + n x (x2 – 1)
4x 2 – 1

1
4x2 – 1 > 0 ⇒ x > ; x < –1/2
–3 –2 8 2
Solution Slot – 1 (Mathematics) Page # 19

x (x2 – 1) > 0 ⇒ x (x + 1) (x – 1) > 0 ⇒ logx (cos 2πx) ≥ 0


x ∈ (–1, 0) ∪ (1, ∞) Case–1 : 0 < x < 1 ⇒ 0 < cos 2πx ≤ 1
⇒ 0 < 2πx < 2πLet t = 2πx
⇒ 0 < t < 2π
–1 1 0 1 1

2 2

π
 1 π 2π
∴ x ∈  − 1, −  ∪ (1, ∞) 3π
 2 2 2

 x  π 3π
(viii) f(x) = log1/ 2   ⇒ 0≤t< ∪ < t ≤ 2π
 x2 − 1 2 2

π 3π
x x x2 − x − 1 ⇒ 0 ≤ 2πx < ∪ < 2πx ≤ 2π
log1/2 ≥0 ⇒ ≤1⇒ ≥0 2 2
x2 − 1 x2 − 1 x2 − 1
1 3
1 − 5  1 + 5  ⇒ 0 ≤ x< ∪ <x≤ 1
, ∞ 4 4
∴ x ∈ (–∞, –1) ∪  , 1 ∪  
  2
 2   
 1 3   1 3 
⇒ x ∈ 0,  ∪  ,1 ∴ x ∈  0,  ∪  ,1
x x  4 4   4 4 
& >0⇒ >0
x −12 ( x + 1)( x − 1) Case–2 : x > 1 ⇒ cos 2πx ≥ 1
⇒ x ∈ (–1, 0) ∪ (1, ∞) ⇒ cos 2πx = 1 ⇒ 2πx = 2nπ ⇒ x = n

1 − 5  1 + 5  1
∴ x∈  , 0 ∪  , ∞ cos x –
  2
 2   2  (xii) f(x) =
6 + 35 x – 6 x 2
1
(ix) f(x) = x2 – | x | + 1
9 – x2 ⇒ 6 + 35x – 6x2 > 0 ; cos x ≥
2
x2 – |x| ≥ 0 9 – x2 ≥ 0
|x| ≤ x2 x2 – 9 < 0 1
⇒ (x – 6) (6x + 1) < 0 ⇒ – <x< 6
|x| ≤ |x|2 –3< x<3 6
|x| ≤ 0 or |x| ≥ 1
x ≤ –1 or x ≥ 1
y=1/2
x= 0
π 5π 2π
3 3

x=–1/6 x=6

–3 –1 0 1 3  −1 π   5π 
∴ x ∈  ,  ∪  , 6
∴ x ∈ (–3, –1] ∪ (1, 3) ∪ {0}  6 3 3 

(x) f(x) = (x2 – 3x – 10)n2 (x – 3) (xiii) f(x) = log1/3 (log4 ([x]2 − 5))
x2 – 3x – 10 ≥ 0 x–3>0 ⇒ log1/3 log4 ([x]2 – 5) ≥ 0
(x – 5) (x + 2) ≥ 0 x> 3 ⇒ 0 < log4 ([x]2 – 5) ≤ 1
x≥ 5 x≤ – 2 n2 (x – 3)= 0 ⇒ 1 < [x]2 – 5 ≤ 4 ⇒ 6 < [x]2 ≤ 9
x – 3 = 1⇒ x = 4 [x] → is always give integer value so it square
also give integer value so,
⇒ [x]2 = 9 ⇒ [x] = ± 3
–2 3 5 ⇒ [x] = 3 ⇒ 3≤x<4
∴ x ∈ [5, ∞) ∪ {4} & [x] = –3 ⇒ –3 < x ≤ –2
∴ x ∈ [–3, –2) ∪ [3, 4)
(xi) f(x) = log x (cos 2πx )
Page # 20 Solution Slot – 1 (Mathematics)

1 1 1 1
(xiv) f(x) = + log(2{x}–5) (x2 – 3x + 10) + +
[ x] 1− | x | 2− | x | + sec(sin x )
D1 D2 D2 D1 : x ∈ R – [0, 1)
D1 → R – [0, 1) D2 : x2 – 3x + 10 > 0 ; 1 – {x} > 0 ; 1 – {x} ≠ 1
D2 : log(2{x}–5) (x2 – 3x + 10) x∈R {x} < 1 [x] ≠ 0
x∈R x≠ Ι
5
& 2{x} – 5 > 0 ⇒ {x} > Not possible D3 : 2 – | x | > 0 ⇒ | x | < 2 ⇒ –2 < x < 2
2
D4 : x ∈ R × × ×
∴ x∈φ –2 –1 0 1 2
(xv) f(x) = logx sin x ∴ x ∈ (–2, –1) ∪ (–1, 0) ∪ (1, 2)
x > 0, x ≠ 1, sin x > 0
& 0 < x < π ⇒ 2nπ < x < 2nx + π (xviii) f(x) = (5 x − 6 − x 2 ) [{n { x }}] +
⇒ 2nπ < x < (2n + 1) π n∈Ι D1
−1
    7 
   (7 x − 5 − 2x ) +  n  2 − x  
2
1   
(xvi) f(x) = log2  −log1/2 1+ 
   xº   D2 D3
  sin 100   D1 : x∈R –I
    
& 5x – 6 – x2 ≥ 0 ⇒ (x – 2) (x – 3) ≤ 0 ⇒ 2 ≤ x ≤ 3
D2 : 7x – 5 – 2x2 ≥ 0 ⇒ 2x2 – 7x + 5 ≤ 0
+ log10 (log10 x ) − log10 ( 4 − log10 x ) − log10 3 5
⇒ (2x – 5) (x – 1) ≤ 0 ⇒ 1≤x≤
⇒ log10 (log10 x) – log10 (4 – log10 x) – log10 3 ≥ 0 2

 log10 x  1 7 
log10  3(4 − log x)  ≥ 0 D3 : ⇒ n  − x  ≤ 0
   7  2 
10 n  − x 
2 
log10 x
⇒ 3( 4 − log x ) ≥1 ⇒ t ≥ 12 – 3t (let log10 x = t) 7 5
10 ⇒ – x≠ 1 ⇒ x≠
2 2
⇒ t≥3 ⇒ log10 x ≥ 3 ⇒ x ≥ 103
& log10 x > 0 ⇒ x > 1 and 4 – log10 x > 0 ⇒ x < 104 7 7
& –x>0 ⇒ x<
2 2
∴ x ∈ [103, 104)
3 4
1 10 10
 5
    ∴ x ∈  2, 
 1   1 
1 2 5
2
3 7
2
 2
– log1/2 1 +  > 0 ⇒ log2  1 +  >0
xº 
 xº  
sin sin  (xix) f(x) = x 2 − 5x + 4 g(x) = x + 3
 100   100 

1 1 f f (x) x2 − 5x + 4
⇒ 1+ xº > 1 ⇒ >0 (x) = =
xπ g g( x ) x+3
sin sin
100 1800 x2 – 5x + 4 ≥ 0 x+3≠ 0
(x – 4) (x – 1) ≥ 0 x ≠ –3
xπ xπ
⇒ sin >0 ⇒ 0< <π x≥ 4 x≤ 1
18000 18000
∴ x ∈ (–∞, –3) (–3, 1] ∪ [4, ∞)
⇒ 0 < x < 18000

Sol.2 (i) y = log [( 2 sin x − cosx) + 3)]


∴ x ∈ [103, 104) 5
3 4
1 10 10 18000
⇒ – 2 ≤ sin x – cos x (m) ≤ 2
1
(xvii) f(x) =
[ x]
+ log1 – {x} (x2 – 3x + 10) ⇒ –2 ≤ 2m≤2 ⇒ 1≤ 2m+3≤5
Solution Slot – 1 (Mathematics) Page # 21

(vi) f(x) = log(cosec x – 1) (2 – [sin x] – [sin x]2)


⇒ log 5 ≤ log 5 ( 2m + 3 ) ≤ log 5 5 cosec x – 1 ≠ 1 ⇒ cosec x ≠ 2

∴ 0 ≤ log 5 ( 2m + 3) ≤ 2 π 5π
x ≠ 2nπ + , 2nπ +
6 6
Range ∈ [0, 2] ; Domain ∈ R
2x π
also cosec x ≠ 1 ⇒ x ≠ 2nπ +
(ii) y= 2 ⇒ y + yx2 – 2x = 0 2
1+ x
also 2 – t – t2 > 0
⇒ yx2 – 2x + y = 0 Domain x ∈ R
where t = [sin x]
⇒ D≥0 ⇒ 4 – 4(y2) ≥ 0
⇒ t2 + t – 2 < 0 ⇒ (t + 2) (t – 1) < 0
⇒ 1 – y2 ≥ 0 ⇒ –1 ≤ y ≤ 1
⇒ –2 < t < 1 ⇒ –2 < [sin x] < 1
x 2 − 3x + 2 ( x − 2)( x − 1) ⇒ –1 ≤ sin x < 1
(iii) f(x) = =
x +x−6 2 ( x + 3)( x − 2) but since cosec x can’t be –ve hence
Domain x ∈ R – {–3, 2} π
5π / 6 2
(1)
π/6
x −1
y= ; x≠2
x+3
0 ≤ sin x < 1 (2n+1)n 2nπ
1 [0] [0]
yx + 3y = x – 1 ; when x = 2, y =
5 (–1)

3y + 1 ∴ x ∈ (2nπ, (2n + 1)π) –


1 
x= ∴ Range y ∈ R –  , 1
1− y 5  π π 5π
{2nπ + , 2nπ + , 2nπ + } where n ∈ I
2 6 6
x
(iv) y=
1+ | x | x+4 −3
(vii) f(x) =
x−5
x
If x ≥ 0 ⇒ y = ⇒ y + yx = x Domain : x + 4 ≥ 0 ⇒ x ∈ [–4, ∞] & x ≠ 5
1+ x
∴ x ∈ [–4, ∞) – {5}
y y
⇒ x= ≥0 ⇒ ≤0 ( x + 4 − 3) ( x + 4 + 3)
1− y y −1 Range : y = f(x) =
( x − 5) ( x + 4 + 3)
∴ 0≤y<1

x y ( x − 5)
If x ≤ 0 ⇒ y = ⇒ x= ≤0 f(x) =
1− x y +1 ( x − 5) ( x + 4 + 3)
∴ y ∈ [–1, 0]
∴ Domain x ∈ R, Range ∈ (–1, 1) 1 1
Lim f(x) = Lim
x →5 x → 5 ( x + 4 + 3) = 6 (x ≠ 5)
(v) y= 2 − x + 1+ x
2–x≥ 0 1+x≥ 0 1
x≤ 2 x ≥ –1 so x ≠ 5, y≠
6
∴ Domain x ∈ [–1, 2]
1
y2 = 2 – x + 1 + x + 2 (2 − x ) (1 + x ) Also f(x) =
x+4 +3
=3+2 (2 − x ) (1 + x )
1 1
⇒0≤ x+4 <∞⇒0< ≤
3 x+4 +3 3
∴ 0≤ (2 − x ) (1 + x ) ≤
2
 1  1 1
⇒ 3≤3+2 (2 − x ) (1 + x ) ≤ 6 so range of y is y ∈  0,  ∪  , 
 6  6 3
⇒ 3 ≤ 3 + 2 (2 − x ) (1 + x ) ≤ 6
Sol.3 |2x + 5| + |2x – 5| = Px + 10
∴ Range y ∈ [ 3 , 6]
Page # 22 Solution Slot – 1 (Mathematics)

5 x
–(2x + 5) + 5 – 2x = – 4x x≤ – x
2 ⇒ y = 2 x −1 ⇒ log2 y =
x −1
⇒ x log2 y – log2 y = x
5 5
2x + 5 + 5 – 2x = 10 – <x< ⇒ x [log2 y – 1] = log2 y
2 2
log y log2 x
5 ⇒ x= ∴ f–1 (x) = log x − 1
2x + 5 + 2x – 5 = 4x x≥ log y − 1 2
2
y=–4x
P = slope y=4x
10 x − 10 − x 102 x − 1
(iii) y= ⇒ y=
P ∈ (–4, 4) – {0}
 5 
 − , 10 
 2 
5 
 , 10 
2 
10 x + 10 − x 102x + 1
⇒ yt + y = t – 1 where t = 102x
Sol.4 f(x) = x ; 0≤ x≤ 1 1+ y 1+ y
=1 ; x>1 ⇒ t= ⇒ 102x =
1− y 1− y
(i) If even then f(–x) = f(x)
f(x) = – x ; –1 ≤ x ≤ 0 1+ y
1 1 1+ x
=1 ; x < –1 ⇒ x= log ∴ f–1 (x) = log
2 1− y 2 1− x
(ii) If odd then f(–x) = – f(x)
f(x) = x ; –1 ≤ x ≤ 0
=–1 ; x < –1 Sol.8 (i) 10x + 10y = 10
⇒ 10 = 10 – 10x
y
⇒ y = log (10 – 10x)
π /2
Sol.5 x ∈ [0, 1] Domains : 10 – 10 > 0 ⇒ 10 > 10x ⇒ x < 1
x

(a) f(sin x) (ii) x + | y | = 2y


⇒ 0 ≤ sin x ≤ 1
π
(2n+1) π
0 2n π If y ≥ 0 then x + y = 2y ⇒ y = x
⇒ 2nπ ≤ x ≤ (2n + 1)π Domain : x ≥ 0
(b) f(2x + 3) ⇒ 0 ≤ 2x + 3 ≤ 1 x
If y < 0 then x – y = 2y ⇒ y =
3 3
⇒ –3 ≤ 2x ≤ –2 ⇒ – ≤ x ≤ –1 Domain : x < 0
2

1  3  Sol.9 (a) f(x) = log (x + 1+ x 2 )


Sol.6 f :  , ∞  →  , ∞  ; f(x) = x2 – x + 1
2  4 
& f(–x) = log (– x + 1+ x 2 )
–1
f(x) = f (x) ⇒ f(x) + f(–x) = log (–x2 + 1 + x2)
f(x) = x ⇒ x2 – x + 1 = x ⇒ f(x) + f(–x) = 0
⇒ x2 – 2x + 1 = 0 ⇒ (x – 1) = 0 ⇒ x = 1 ⇒ f(–x) = – f(x) ∴ odd function

x (a x + 1)
Sol.7 (i) f(x) = n (x + x2 + 1 ) (b) f(x) =
ax − 1
y = n (x + x 2 + 1 ) ⇒ ey = x + x2 + 1
x (a − x + 1) x (1 + a x )
& f(–x) = – =–
⇒ e2y = x2 + x2 + 1 + 2x x 2 + 1 (a − x − 1) (1 − a x )
⇒ e2y = 2x2 + 1 + 2x (ey – x)
⇒ e2y = 2x2 + 1 + 2xey – 2x2 x (1 + a x )
= ∴ even function
2y (a x − 1)
e −1 1 y
⇒ 2x = y ⇒ x= (e – e–y) (c) f(x) = sin x + cos x
e 2
& f(–x) = – sin x + cos x
ey − e−y f(–x) ≠ f(x) None
∴ f–1 (y) = (d) f(x) = x sin2 x – x3
2
& f(–x) = – x sin2 x + x3 = – f(x) odd
x (e) f(x) = sin x – cos x
(ii) f(x) = 2 x −1 & f(–x) = – sin x – cos x None
Solution Slot – 1 (Mathematics) Page # 23

(1 + 2 x )2
(f) f(x) =
2x 1
Sol.11 f(x + a) = + f − f2
2
(1 + 2− x )2 (1 + 2 x )2
& f(–x) = −x = even replace x → x + a [where f’ = f(x + a)]
2 2x
1 1
xx f(x + 2a) =
2
+ f'− f'2 = 2 + f '(1− f')
(g) f(x) = x + +1
e −1 2
1 1 21 2
x −x =
2
+ 2 + f −f 2 − f −f 
& f(–x) = − x – +1   
e −1 2
1 1
x x x ex x = + − f + f2
= −x – +1= x – +1 2 4
1− e 2 e −1 2
1 1 1 1
f(x + 2a) = + f− = +f–
x ex x x ex − x ex + x x 2 2 2 2
= x – x + +1 = + +1
e −1 2 ex − 1 2
f(x + 2a) = f(x)
x x
f(–x) = x + +1 even Sol.12 f(1) = 2 ; f(2) = 8
e −1 2 f(x + y) – k xy = f(x) + 2y2 Put y = 1, k = 4
(h) f(x) = [(x + 1)2]1/3 + [(x – 1)2]1/3 f(1 + y) = 4y + f(1) + 2y = 4y + 2 + 2y2
2

& f(–x) = [(–x + 1)2]1/3 + [(–x –1)2]1/3 = 2 (y2 + 2y + 1) = 2(y + 1)2


= [(x – 1)2]1/3 + [(x + 1)2]1/3
f(–x) = f(x) even  1  2
f(x) = 2x2 ; f(x + y) = 2(x + y)2 ; f   =
 x + y  ( x + y )2
Sol.10 (i) f(f(x)) . (1 + f(x)) = – f(x)
 1 
f (x) −x f(x + y) f   = 4 = k
⇒ f(f(x)) = – ⇒ f(x) = x+y
1 + f (x) 1+ x

−3 −3
f(3) = =  1
1+ 3 4  − |n {x}| |n {x}|
Sol.13 f(x) =  e − {x} ; where ever it exists
(ii) f(x + f(x)) = 4f(x) ; f(1) = 4 
f(1 + f(1)) = 4f(1) ⇒ f(5) = 4.4 ⇒ f(5) = 16  {x} ; otherwise, then

f(5 + f(5)) = 4 + (5) ⇒ f(21) = 4.16 = 64 For first definition of f(x)


(iii) f : R+ → R+ ; [f(xy)]2 = x [f(y)]2 as 0 ≤ {x} < 1 But {x} ≠ 0 ⇒ x ∉ I
x = 25 , y = 2
(f(50))2 = 25 (f(2))2 ⇒ f(50) = 5 . f(2) = 5 .6 = 30  1
(iv) f(x + y) = x + f(y) f(0) = 2  − |n {x}| |n {x}|
f(x) =  e − {x} ; x∉I
y = 0, x = 2 
f(2) = 2 + f(0) = 4  {x} ; x∈I
f(2 + 2) = 2 + f(2) = 2 + 4 = 6
f(2 + 4) = 2 + f(4) = 2 + 2 + 4 = 8 −1
f(2 + 6) = 2 + f(6) = 2 + 2 + 2 + 4 = 10 − − n { x } n { x }
f(x) = e − {x}
f(100) = 102
(v) f(3) = 1 1
n { x }
f(3x) = x + f(3x – 3) = e–t – e t (where t = − n { x } )
x = 1 f(3) = 1 + f(0) ⇒ f(0) = 0 –t –t
=e –e =0
x = 2 f(6) = 2 + f(3) = 2 + 1
f(x) = 0
x = 3 f(9) = 3 + f(6) = 3 + 2 + 1
f(x) = 0 is only the function which is odd as well
x = 4 f(12) = 4 + f(a) = 4 + 3 + 2 + 1 as even.
f(3 × 100) = 100 + 99 + 98 +.......+ 1
= 5050
Page # 24 Solution Slot – 1 (Mathematics)

9x  4
Sol.14 f(x) = x ∈ 1, 
x
9 +3  3
a=1;b=4;c=3
 1  91/ 2006 a + b + c + abc = 1 + 4 + 3 + 12 = 20
f  =
 2006  91/ 2006 + 3
 1− x 
2005 Sol.16 f : R → R ; f   =x
 2005  9 2006 9 . 9 −1/ 2006  1+ x 
f  = =
 2006  2005 9 . 9 −1/ 2006 + 3 1− x 1− t
9 2006 + 3 Put =t ⇒ x=
1+ x 1+ t
9 3 1− t 1− x
= = ∴ f(t) = ⇒ f(x) =
9 + 3 . 91/ 2006 3+9 1/2006
1+ t 1+ x

 1− x 
 1   2005   1−  
f  + f  =1  1− x   1+ x 
 2006   2006  (a) f[f(x)] = f   = =x
  1+ x   1− x 
  1+  
 2   2004    1+ x 
f  + f  = 1  ⇒ 1002
 2006   2006  
   1 1 − 1/ x x −1
 1002   1004   (b) f  = = = – f(x)
f  + f  =1  n 1 + 1/ x x +1
 2006   2006  
1 − ( − x − 2) 1+ x + 2
(c) f(–x –2) = =
1 + ( − x − 2) 1− x − 2
 1003  3 1
f  = =
 2006  3+3 2 1 + 2x − x + 2  1− x 
= = –  – 2 = – f(x) – 2
sum = 1002 + 0.5 = 1002 . 5 − (1 + x )  1+ x 

3  4  1
Sol.15   +   = 5 ; x> 0 Sol.17 f(x) = max  x,  x> 0
x x  x
= max (a, b)
3  4 (0,1) (1,1)
Case-I :   = 1 & x = 4
x    1
g(x) = f(x). f   (1,0)
x x>0
3 4
1≤ <2 4≤ <5
x x 1
f(x) = ;0< x≤ 1
x
3 4
< x≤ 3 < x≤ 1 = x ; x>1
2 5
x∈φ  1 1
f  = x ; 0< ≤ 1⇒x≥ 1
x x
3  4
Case–II :   = 2 & x = 3
x   1 1
= ; > 1 ⇒ x< 1
x x
3 4
2≤ <3 3≤ <4
x x  1
g(x) = f(x) . f   = x2 ; x>1
3 4 x
1< x≤ 1< x≤
2 3
1
= ; 0 < x< 1
x2
Solution Slot – 1 (Mathematics) Page # 25

Sol.18 True False False (d) P(x) = x6 + ax5 + bx4 + cx3 + dx2 + ex + f
(i) If f(x) = 1 f(y) ≠ 1 f(z) ≠ 2 possible p(x) will be
f(x) = 1 f(y) = 1 f(z) = 2 P(x) = (x – 1)(x – 2) (x – 3)(x – 4)(x – 5)(x – 6) + x
But function is one-one so not possible P(7) = 6! + 7 = 720 + 7 = 727
False True False (e) f(x) = a sin x + b(x)1/3 + 4
(ii) If f(x) = 1 f(y) ≠ 1 f(z) ≠ 2 f(log 10 (log 3 10))
f(x) = 3 f(y) = 1 f(z) = 2 = a sin (log10 (log310)) + b (log10 (log310))1/3 + 4 = 5
f–1 (1) = y = a sin (log10 (log103)–1) + b (log10(log103)–1)1/3 + 4 = 5
False False True = – a sin (log10 (log10 3)) – b (log10 log10 3) = 1
(iii) If f(x) = 1 f(y) ≠ 1 f(z) ≠ 2 = – (M) = 1 ⇒ M = –1
f(x) = 2 f(y) = 1 f(z) = 3 f(log10 log10 3) = a sin (log10 log10 3) + b(log10 log10 3) + 4
f–1 (1) = y =–1+4=3

f (1)  1  1
Sol.19 (a) f(1) + f(2) = 4f(2) ⇒ f(2) =
3 Sol.20  x −   x + 2  is prime So
 2  
f (1)
f(1) + f(2) + f(3) = 3 ⇒ f(3) = 1 1
6 Case-I :1 ≤ x – <2 & 2 ≤ x+ <3
2 2
f (1) 3 5 3 5
f(1) + f(2) + f(3) + f(4) = 16 ⇒ f(4) = ≤x< ≤x<
10 2 2 2 2
2 f (1)
⇒ f(n) =  3 5
n(n + 1) so x∈  , 
 2 2
2 f (1)
f(2004) = 1 1
2004 ×2005 Case-II : –2 ≤ x – < –1 & –1 ≤ x + <0
2 2
1 3 1 3 1
= (as f(1) = 2005) – ≤ x<– – ≤x<–
1002 2 2 2 2
(b) Given a – b = 2
 3 − 1
x 2 + ax − x 2 + bx < L x ∈ − , 
 2 2 

(a − b ) x  3 − 1 3 5 
2 2
<L Final x ∈ − ,  ∪  , 
x + ax + x + bx  2 2  2 2 

2x 3 −1 3 5
<L x1 = ; x2 = ; x3 = ; x4 =
2
x + ax + x + bx 2 2 2 2 2

9 + 1 + 9 + 25
2 x12 + x22 + x32 + x42 = = 11
1
<L 4
a b
1+ + 1+
x x Sol.21 P(x) = (x – 1) Q1 + 1 ⇒ P(1) = 1
L > 1 so min. value of L = 1 P(x) = (x – 4) Q2 + 10 ⇒ P(4) = 10
(c) f(f(x)) = (x2 + kx)2 + k(x2 + kx) = 0 P(x) = (x – 1)(x – 4) Q3 + (ax + b)
(x2 + kx) {x2 + kx + k} = 0 P(1) = 1 ⇒ a + b = 1
for same solution x2 + kx + k = 0 must not P(4) = 10 ⇒ 4a + b = 10
have any real roots a = 3 ; b = –2
D<0 r(x) = 3x – 2 ⇒ r(2006) = 6016
k2 – 4k < 0
k ∈ (0, 4)
check for end values of k {k = 0 is possible}
so k ∈ [0, 4)
Page # 26 Solution Slot – 1 (Mathematics)

f (x) − 5
Sol.22 f(x + T) = ....(i)
f (x) − 3
Replace x by x + T
f ( x + T ) − 5 2f ( x ) − 5
f(x + 2T) = = ....(ii) (from (i))
f ( x + T) − 3 f ( x) − 2
Replace x by x + 2T
2f ( x + 2T ) − 5
f(x + 4T) = = f(x) (from (ii))
f ( x + 2T ) − 2
f(x) is periodic with period 4T

Sol.23 f(x) = x135 + x125 – x115 + x5 + 1


x135 + x125 – x115 + x5 + 1 = (x3 – x) Q(x) + (ax2 + bx + c)
put x = 0 ⇒ c = 1
put x = 1 ⇒ a + b = 2
put x = –1 ⇒ a – b = –2
a = 0, b = 2
g(x) = 2x + 1 ⇒ g(10) = 21

Sol.24 Area of shaded region = area of arc OAB – area


of ∆OAB

πr 2 1 1
= r2 (π – 2x) – r2 sin 2x
3 2 2

2π A
π – 2x – sin 2x =
3
X 2x
r
π O
f(2x) = 2x + sin 2x – X r
3
B C
π
f(x) = x + sin x –
3
Solution Slot – 1 (Mathematics) Page # 27

EXERCISE – V HINTS & SOLUTIONS


Sol.1 B
f : [1, ∞) → [1, ∞) ; y = f(x) ⇒ x = f–1 (y) x + x2 − 4
∴ f (x) =
–1
y = 2x (x – 1) ⇒ log2 y = x (x – 1) 2

(c) D
1 ± 1 + 4 log2 y
⇒ x2 – x – log2 y = 0 ⇒ x =
2 log2 ( x + 3) log2 ( x + 3)
f(x) = =
2
x + 3x + 2 ( x + 1)( x + 2)
1 + 1 + 4 log2 y
x should be positive so x = x + 3 > 0 ⇒ x > –3 & x ≠ –1, –2
2
(d) A
E → F ; Number of functions = 24 = 16
1 + 1 + 4 log2 y
∴ f (y) =
–1
Each element of E is connect with element
2
1 of F so 2 is left.
when each element of E is connect with
1 + 1 + 4 log2 x
⇒ f–1 (x) = Element 2 of F so 1 is left
2
so function is onto in two situation
so number of onto functions = 16 – 2 = 14
Sol.2 D (e) D
2y + 2x = 2 ⇒ 2y = 2 – 2x ⇒ y = log (2 – 2x)
∴ 2 – 2x > 0 ⇒ 2x < 2 ⇒ x < 1 αx
f(x) =
x +1

Sol.3 x = {1, 2, 3, 4} ; f : x → x
 αx 
α 
1 1 1 1 1 1
 x + 1 α2x
2 2 2 2 2 2 f(f(x)) = =x ⇒ =x
3 3 3 3 3 3 αx αx + x + 1
4 4 4 4 4 4 +1
x +1

⇒ α2 = (α + 1)x + 1
Sol.4 (a) B
∴ α2 = 1 ⇒ α = ±1, α + 1 = 0 ⇒ α = –1
so α = –1
 −1 x<0

g(x) = 1 + x – [x] ; f(x) =  0 x=0
 1 x>0 Sol.5 (a) D
clearly g(x) will be inverse of f(x)
we know that x – [x] = {x} f(x) = y ⇒ x = f–1 (y)
∴ g(x) = 1 + {x} > 1 ∀ x ∈ R
y = (x + 1)2 ⇒ x + 1 = ± y ⇒ x = –1 ± y
or f[g(x)] = 1 from definition of f(x) = 1, x > 0
(b) A
∵ x ≥ –1 ⇒ x = –1 + y
f : [1, ∞) → [2, ∞) ; f(x) = y ⇒ x = f–1 (y)

⇒ f–1 (y) = –1 + y ∴ f–1 (x) = – 1 + x


1 y ± y2 − 4
y = x+ ⇒ x2 – xy + 1 = 0 ⇒ x = (b) A
x 2
f(x) = 2x + sin x
x should be greater than 1 so
f ’(x) = 2 + cos x > 0 ∀ x ∈ R

y + y2 − 4 Also f(x) → ∞ as x → ∞ & f(x) → –∞ as x → –∞


y + y2 − 4
x= ⇒ f–1 (y) = Thus f(x) is one-one and onto
2 2
Page # 28 Solution Slot – 1 (Mathematics)

Sol.6 (a) A 11. B


f(X) = 2X3 – 15X2 + 36X + 1
1+ x − x 1 f'(X) = 6X2 – 30X + 36
f ’(x) = 2 2 =
(1 + x ) (1 + x 2 )2 = 6 (X2 – 5X + 6)
= 6(x – 3) (x – 2)
⇒ f is increasing ⇒ f is one-one
Range of f = [0, 1) ⇒ f is not onto
(b) D (0,29)
(0,28)
(0,1)
x2 + x + 2
y= ⇒ x2 + x + 2 = x2y + xy + y
x2 + x + 1 (2,0) (3,0)

⇒ x2(1 – y) + x(1 – y) + 2 – y = 0 (∴ y ≠ 1) f(0) = 1


D ≥ 0 ⇒ (1 – y)2 – 4(1 – y) (2 – y) ≥ 0 f(2) = 2 × 8 – 15 × 4 + 36 × 2 +1 = 29
⇒ 1 + y2 – 2y – 4(2 – 3y + y2) ≥ 0 f(3) = 2 × 27 – 15 × 9 + 36 × 3 + 1 = 28
⇒ 3y2 – 10y + 7 ≤ 0 onto but not one one so 'B'.

7 7 12. A,B
⇒ 1≤y≤ ∴ 1<y≤
3 3
2 cos2 θ 1 + cos 2θ
f(cos 4θ) = =
2 cos2 θ − 1 cos 2θ
Sol.7 g(f(x)) = (sin x + cos x)2 – 1 = sin 2x
–1 ≤ sin 2x ≤ 1 1
⇒ 2 cos 2θ − 1 =
2
π 3
π π −
2
– ≤ 2x ≤ π
2 2 2 2 2
⇒ cos2 2θ = ⇒ cos 2θ = ±
3 3
π π
– ≤ x≤
4 4 1 3
f  =1±
3
  2
Sol.8 A

 x ; x ∈ irrational
Let h(x) = f(x) – g(x) = 
− x ; x ∈ rational

⇒ the function h(x) is one-one and onto.

Sol.9 D
Each element in either (A) or (B) or neither.
4
∴ Total ways = 3 = 81
A = B iff A = B = φ (1 case)
otherwise A & B are interchangeable
∴ n = 1 + (81 – 1) / 2 = 41

10. A
fogogof(x) = sin2 (sin x2)
gogof(x) = sin (sin x2)
on solving, sin (sin x2) = 0 or1
⇒ x=± nπ ; n ∈ {0, 1, 2 ......}

You might also like